You are on page 1of 62

1.

Content
1. Content.....................................................................................................................................1
2. Protection of Women from Domestic Violence ACT, 2005? Explain the reliefs provided
under The Protection of Women under Domestic Violence ACT, 2005.........................................3
3. Define Maintenance. ls there any provision under Islamic LAW for the maintenance of
wife? Explain with cases.................................................................................................................4
4. Discuss in detail the grounds available for a Muslim Woman under the Dissolution of
Muslim Marriage ACT, 1939..........................................................................................................6
5. Describe the procedure of solemnization of marriages of Christians as per the Indian
Christian Marriage ACT..................................................................................................................8
6. A short note on Family Courts ACT......................................................................................10
7. Write a Note on Sapinda and Prohibited relationships...........................................................12
Provisions Concerning Prohibited Relationship Degrees..........................................................13
8. Write a Note on Khula and Mubaarat....................................................................................13
9. Write a Note on Iddat.............................................................................................................14
10. Explain the provisions of The Parsi Marriage and Divorce ACT regarding a divorce..........15
11. Critically discuss the grounds of Divorce Available to Only wife under the Hindu Marriage
ACT, 1955.....................................................................................................................................16
12. Explain the provisions of Hindu Marriage ACT 1955, under which divorce can be claimed
by both the spouses........................................................................................................................19
13. " Right of pronouncing a triple talaq conferred by religion on a Muslim Man has now been
curtailed.” Critically discuss..........................................................................................................21
14. Discuss the provisions of the Hindu Adoption and Maintenance ACT regarding claim of
maintenance by wife, parents and children....................................................................................23
15. What are the Sources and schools of Hindu Law?.................................................................26
16. Discuss Cruelty as grounds for divorce..................................................................................28
17. Explain" Dower". Discuss the remedies of a Muslim wife when dower is not paid to her.. .29
18. What is the difference between Judicial Separation and Divorce?........................................32
19. Write a Note on Nullity of Marriage......................................................................................35
Under Hindu Law:.....................................................................................................................35
Nullity under Muslim personal law:..........................................................................................35
Nullity under Christian law:.......................................................................................................36
20. Write a Note on Irretrievable breakdown of marriage...........................................................37

1|Page
21. Discuss the constitutional validity of SECTION 9 of Hindu Marriage ACT, 1955 relating to
Restitution of Conjugal Rights......................................................................................................39
22. "The remedy of Divorce by mutual consent is recognized under every personal law".
Discuss...........................................................................................................................................43
Mubarat Khulla..........................................................................................................................43
Khula's (At the Request of Wife)...............................................................................................43
Mubarat (Mutual Release).........................................................................................................44
1872 Indian Christian Marriage ACT........................................................................................44
The 1954 Special Marriage ACT...............................................................................................44
23. Difference between void and voidable marriage under HMA and legitimacy of children born
out of void marriages.....................................................................................................................45
24. "SECTION 125 of Criminal Procedure Code gives rights of claiming maintenance to wives,
children and parents irrespective of the religion which they profess" Comment..........................47
25. OPTIONAL-Extra..................................................................................................................49
26. Registration of marriage, Seema v ashwini kumar,................................................................49
27. Conditions and Procedure of registration of marriage under Special marriage act, 1954......50
Conditions for Marriage Registration Under the Special Marriage Act....................................50
Procedure...................................................................................................................................51
28. Solemnization of marriage under Special marriage act, 1954................................................51
Solemnization of Marriage.........................................................................................................51
Procedure for solemnisation of marriage...................................................................................51
29. Remedies available to a woman when dower is not given.....................................................52
30. Restitution of conjugal rights under muslim, Christian law and the special marriage act.....53
Restitution of Conjugal Rights under Muslim Law...................................................................53
When can Restitution of Conjugal Rights be refused under Muslim Law?...............................54
Restitution of Conjugal rights under Christian Law..................................................................54
Restitution of Conjugal Rights under Special Marriage Act, 1954...........................................55
31. Definition of aggrieved person and domestic relationship under law....................................55
32. Latifi v. Union of India...........................................................................................................55
33. What are the prohibitions on muslim marriages....................................................................57
Absolute Prohibition..................................................................................................................58
Relative Prohibition...................................................................................................................58
34. Doctrine of factum valet.........................................................................................................60

2|Page
2. Protection of Women from Domestic Violence ACT, 2005?
Explain the reliefs provided under The Protection of Women
under Domestic Violence ACT, 2005.

Domestic violence is violence or some sort of abuse involving mental, physical and sexual abuse
in a domestic environment, such as in marriage or cohabitation also widely known as intimate
partner violence. It is a regular occurrence around the globe but in India, it is rampant here since
the concerns of dowry, male domination and residing in joint family set ups are the norm here.
Therefore, these circumstances have an extra part in making the women, victims of domestic
abuse.
The PWDVA authorizes complaints of domestic abuse to be submitted by the aggrieved victims
themselves, protection officials or service providers.
 The purpose of the legislation was to offer enough protection to women who are victims of
domestic abuse of any type, and it would be invalidated if the ACT was not properly and
effectively implemented owing to insufficient protection officers and protection officers
who had the resources and environment required to accomplish their duties under the ACT.
 Acknowledging that a person requires guidance with legal processes and other methods of
aid, the PWDVA promotes Protection Officers to be established and recognises the role of
Service Providers in providing medical, housing, legal, counselling and other support
services.
 The Protection Officer is the person responsible for supporting women in making use of
such services, as well as assisting them in gaining the right order under the ACT. Upon
receipt of a domestic violence accusation, the protection officer must assist the Magistrate in
the performance of his obligations under this ACT by reporting a domestic violence event to
the Magistrate in the form and manner indicated, and transmitting copies to the police
officer in charge of the police station within the borders of the local jurisdiction where
domestic violence is the cause.
 An application shall be in the form and manner submitted to the Magistrate, if the aggrieved
party so wants, requesting relief for the issuing of an order of defines. He shall guarantee
that legal help is offered to the grieved person under the 1987 Legal Services Authority
ACT, and make accessible the required form in which a complaint is to be made available
free of charge. He must preserve a list of all service providers giving legal aid or treatment,
shelter houses and medical services in a specific region within the Magistrate’s authority.
3|Page
He must make a safe shelter home available if the grieving person so requires and transmit a
copy of his report to the police station and the Magistrate having jurisdiction in the area
where the shelter is situated. He will quickly evaluate the grieving lady if she has sustained
bodily harm and transmit a copy of the medical report to the police station and the Judge
with power in the region where the domestic violence is reported to have happened.
 In line with the process specified by the Code of Criminal Procedure, 1973, he shall ensure
that the order for monetary relief under SECTION 20 is complied with and implemented.
The Security Officer shall be under the Magistrate’s authority and control and shall
discharge the responsibilities laid on him by the Magistrate and the Government by, or
under, this ACT.
 SECTIONs 18 through SECTION 22 establish numerous types of redress for victims of
domestic abuse, such as protection orders, monetary relief, custody order, residence decrees,
indemnity order. The following:
o Whilst SECTION 23 allows for temporary ex-parte orders, SECTION 26 and
SECTION 28 are significant provisions which may be fully implemented to secure
relief in any court in civil, criminal or family issues and also to set down their own
processes to deal with PWDVA claims. Several of the rules and processes for
obtaining relief orders are described in SECTIONs 12, 13, 14, 15 and 16 of the
ACT.
o The magistrate shall fix the date of the hearing no later than three days after
receiving the application, and the matter should be disposed of within 60 days of the
date of the first hearing, when applications to the magistrates are allowed under
SECTION 12.
o SECTION13 indicates that the Security Officer shall be notified of the hearing
within two days and that a Statement of Service should be provided by the Staff
Officer.
o SECTION 14 states that any party may be instructed by the magistrate if SECTION
15 makes it plain that the magistrate is entitled to employ a welfare expert’s
assistance to help him/her, preferably with a woman, and SECTION 16 says that the
proceedings may occur on camera.

3. Define Maintenance. ls there any provision under Islamic


LAW for the maintenance of wife? Explain with cases.

According to Muslim law, a husband is obligated to support his wife and family, and the word
maintenance refers to the sum he is required to pay. Under Muslim Law, maintenance is referred
to as nafaqa, and it includes food, clothing, and shelter. Notwithstanding having the resources to
support herself, the woman has the right to maintenance from her spouse. Furthermore, the
marriage agreement may provide for the payment of specific payments by the spouse, and in the
existence of them, the husband is obligated to pay them to the wife. These allowances are known
as kharch-e-pandan, guzara, mewa khore, and so on. This is a legal right that may be asserted.

4|Page
 Maintenance under the CrPC – Pre & Post 1973: Originally, the CrPC (formerly under
SECTION 488) stated that only a wife is eligible to husband's support. The husbands
asserted that once a marriage is dissolved, a woman no longer qualifies as a wife and thus is
not eligible to maintenance. In order to close this gap, an amendment was passed in 1973,
which declared that a divorcee was entitled to maintenance until she remarried under
SECTION 125. Because this clause is secular in character, it extends to all women, notably
Muslim women.
o A woman is only allowed to maintenance under Muslim Personal LAW until the
conclusion of the Iddat period. Iddat is the time during which the couples'
cohabitation ends; when iddat expires, the spouses are divorced. The iddat period
lasts three menstrual periods or 3 lunar months; in the event of pregnant women, the
iddat period lasts until the moment of birth. As a result, we can notice a direct clash,
since CrPC does not respect iddat period and maintenance extends beyond it.
o In Mohammed Haneefa V. Mariam Bi, the Court declared that where personal LAW
and the CrPC conflict, personal LAW takes precedence. The Supreme Court agreed
with this viewpoint in Saira Bano V. A.M Abdul Gafoor. generated a lot of
consternation in the lawmakers to address this issue, SECTION 127(3) (b) was
inserted, which states that if a divorced woman gets an amount owing to
communal, conventional or personal laws, the judge may annul any decision for
maintenance in her favour.
o In Bai Tahira VS. Ali Hussain Fissalli Chothia, it was decided that fee of "illusory
sums" based on Muslim personal laws should be regarded to reduce the amount of
maintenance payable by the partner, but this does not absolve the spouse of liability
because every lady, regardless of religion, is entitled to maintenance. Except where
the aggregate sum prescribed by tradition is more than enough to replace support, the
divorced woman has this entitlement.
o In Fuzlunbi V. K Khader Vali, the Supreme Court added a new condition. The
amount based on Muslim LAW must be almost equivalent to the divorcee's monthly
support, necessary until her remarriage or death, with the explicit objective of
substituting the maintenance.
o Under pressure from Islamist organizations, the Rajeev Gandhi administration tried to
overturn the ruling, and in an effort to do so, it created The Muslim Women
(Protection of Rights on Divorce) ACT, 1986. This ACT was one of the most
contentious pieces of legislation ever adopted. The pertinent statutory provisions are
SECTIONs 3(1)(a) and 4(1), which assert that the ex-husband needs to provide "a
reasonable and fair provision" and maintenance within the iddat period and that if she
is incapable to sustain herself after the iddat period, she can presume maintenance
from her family members and if they cannot pay, she can claim from the Wakf Board
as per S.4(2).
o The Daniel Latifi decision essentially reaffirmed the grounds established in the Shah
Bano case, namely that a husband's obligation to support his wife does not stop with
the iddat period. Nevertheless, it characterized this concept as a reflection on the
ACT that was created in response to the Shah Bano case, rather than as a violation of

5|Page
that ACT, Furthermore, the ACT is compatible with SECTION 125 of the CrPC,
thus there is no contradiction.
o As a result, the legal view is that the ACT 's provisions stem primarily from the
principles established in the Shah Bano case. This has not altered, and it continues to
control problems pertaining to the maintenance of Muslim women upon the
breakdown of marriage.
 The Current Position
o The Supreme Court reiterated the notion in Iqbal Bano V. State of Uttar Pradesh. In
the case, the court maintained the stance that divorced women are qualified for
maintenance beyond the Iddat period and ruled that the ACT 's provisions do not
violate ARTICLEs 14, 15, and 21 of the Indian Constitution.
o The court went on to say that "right under SECTION 125 of Cr. P.C. eradicates only
when she gets "fair or reasonable" compensation under Sec. 3 of the Muslim Women
ACT." The woman will be entitled to support under SECTION 125 of the CrPC.
until the husband fulfils his commitment under SECTION 3 of the Muslim Women
(Protection of Rights on Divorce) ACT, 1986.
o This was reaffirmed in the latest Shabana Bano V. Imran Khan decision, which said
that after the expiration of iddat, a divorced Muslim woman may claim support under
S.125CrPc as far as she does not remarry.
o As a result, the position established in the Daniel Latifi situation remains the last
word and has not changed.

4. Discuss in detail the grounds available for a Muslim Woman


under the Dissolution of Muslim Marriage ACT, 1939

 A woman married under Muslim LAW has the right to seek a divorce decision for the
dissolution of her marriage for any of the following reasons:
o That the husband's whereabouts have been unknown for four years: if the husband has
been gone for four years, the woman may submit a petition for the dissolution of her
marriage.
 If the wife or any other person who is assumed to have information of the
husband is unable to find the husband, he is said to be absent. SECTION 3
requires a woman who files a petition for divorce on this reason to disclose the
names and addresses of all people who would have been the husband's lawful
heirs upon his death. The court sends summonses to all such individuals,
requesting that they appear in court and say whether or not they have any
information of the missing spouse.
 If no one knows, the court issues an order to that effect, which becomes
effective after six months. If the husband returns before the expiration date,
the court will vacate the ruling and the marriage will not be dissolved.

6|Page
o That the husband has ignored or failed to pay for her support for a period of two
years: every husband has a legal responsibility to maintain his wife, and if he does
not, the woman may seek divorce on this basis.
 A husband may not maintain his wife because he disregards her or because he
lacks the financial wherewithal to do so. The outcome would be the same in
both circumstances. The husband's commitment to support his wife is
contingent on the woman's own fulfilment of marriage responsibilities.
 As a result, if the woman lives apart without a legitimate justification, she is
not entitled to a judicial divorce based on her husband's failure to support her
since her own behaviour disentitles her to maintenance under Muslim law.
o That the husband has been condemned to incarceration for seven years or more: the
wife's entitlement to judicial divorce on this reason commences on the day the
sentence becomes final.
 As a result, the decree may be issued in her favour only after the husband's
appeal deadline has expired or the husband's appeal has been rejected by the
ultimate court.
o That the husband has failed to discharge his marital responsibilities for three years
without justifiable cause: the ACT defines ‘marital duties of the husband'. Under
Muslim law, the husband has various marital duties. However, for the purposes of this
SECTION, only those conjugal responsibilities that are not covered by any of the
provisions in SECTION 2 of this ACT may be considered.
o That the husband was infertile at the time of marriage and remains to be so: the
woman must establish that the husband was incapable at the time of marriage and
remains to be incapable until the filing of the ACT ion in order to get a divorce
decision on this basis.
 Before issuing a divorce order on this cause, the court is required to let the
husband one year to enhance his potency if he applies for it.
 If the husband does not file such an application, the court must issue the
decree as soon as possible.
 In Gul Mohd. Khan V. Hasina, the wife brought a divorce ACT ion on the
grounds of impotency. The spouse petitioned the court for an injunction
requiring him to prove his incapacity. The court granted him the opportunity
to demonstrate his prowess.
o If the spouse has been mad for two years or is afflicted from leprosy or a severe
veneral disease: the husband's lunacy must have existed for two or more years prior to
the filing of the complaint. However, this statute does not define whether the mental
illness must be treatable or incurable. Leprosy may be white or black, and it can cause
the skin to wither. It might be treatable or fatal. Veneral illness is a sexual organ
disease. According to the ACT, this condition must be incurable. It may be of any
length. Furthermore, even if the woman infected the husband with this ailment, she is
entitled to divorce on this basis.

7|Page
o That she, having received in marriage by her father or other protector before the age
of fifteen, renounced the marriage before the age of eighteen, provided that the
marriage was not consummated;
 In Syed Ziauddin V. Parvez Sultana, Parvez Sultana was a science graduate who wished to
enrol in a medical school. She needs funds for her education. Syed Ziaudddin offered to give
her money in exchange for marrying him. She did it. Later, she filed for divorce due to the
husband's failure to fulfil a commitment. The judge awarded her divorce based on cruelty. As
a result, we can observe the court's stance toward giving a broader connotation to the term
cruelty.
 In Aboobacker V. Mamu koya, the husband forced his wife to wear a sari and watch movies.
The wife rejected because, in her opinion, it was against the Islamic way of life. She filed for
divorce citing mental mistreatment. The Kerela High Court ruled that the husband's
behaviour could not be considered harsh since just deviating from the norms of stifling
orthodoxy doesn't really constitute un-Islamic behaviour.
 The Allahabad High Court observed in Tiwari V. Asghari that Indian LAW does not
acknowledge multiple kinds of cruelty such as "Muslim cruelty," "Hindu cruelty," and so on,
and that the exam of cruelty is predicated on universal and humanist norms; that is, conduct
of the husband that would cause such psychobiological pain as to jeopardize the wife's safety
or health.
 Cruelty by spouse- If the husband is cruel to his wife, she may go to court and get a judicial
separation decision on the same grounds. Physical violence, making defamatory claims
hurting her reputation, are some of the instances in which reasons for cruelty might be
established. Husband has more than one wife and does not treat them as equals, forcing her to
live an immoral life, preventing her from practicing her faith.

5. Describe the procedure of solemnization of marriages of


Christians as per the Indian Christian Marriage ACT.

There are specific formalities that must be followed in every Indian marriage in all groups. So,
for the solemnization of marriage, there are norms and processes in Indian Christian Marriage as
well.
 Time to solemnize Christian marriage.
Christian marriages may only be solemnized between the hours of 6 a.m. and 7 p.m. If a
marriage is solemnized outside of the hours of 6 a.m. and 7 p.m., or in the absence of two
witnesses, the individual will be penalized with jail for up to three years and a fine.
o However, there are certain exceptions. 
 A specific license should be issued. Authorization from a Church Clergyman to the
Clergyman who will solemnize the union at any time other than 6 a.m. and 7 p.m.
 Marriage is solemnized by a Church of Scotland pastor in accordance with the
Church's norms, rituals, and ceremonies.

8|Page
 Marriage may be solemnized by a Clergyman of the Church of Rome who has been
granted a general or special permission.
 Location of Christian Marriage Solemnization
The marriage may only be solemnized in a church where worship is generally conducted
in the form of the Church of England, as authorized by Sec. 11 to Clergyman of the
church. Alternatively, if there is no Church within 5 miles, the priest is granted a special
permission to solemnize the union at any other location. SECTIONs 12 to 26 of the
Indian Christian Marriage ACT deal with the procedure for solemnizing Christian
marriages.
 Furthermore, 
o Sec. 12 deals with marriage notice: Either party is obliged to provide written notice, in
the form included in the first schedule of the ACT, advising the responsible Minister of
the details of the other party to whom they want to marry. The following information
should be included in the notice: NAME, SURNAME, PROFESSION, AND
RESIDENCE of both marriage parties.
o Sec. 13 deals with the publishing of Notice; there is a Minister concern if the couple
wish to marry in the Church or have their marriage solemnized in a Church.
o SECTION 14: Notice of impending marriage in a private residence. If the marriage is to
be solemnized in a private house, the Minister of Religion should transmit the notice
provided in SECTION 12 to the Marriage Registrar of the district, who shall fasten the
same to some visible location in his own office.
o Sec. 15 deals with situations in which either party to the marriage is a minor; in such
cases, the Ministry is obligated to transmit a copy of the notification to the marriage
registrar or senior marriage registrar of the relevant district.
o Sec. 16 deals with the method for receiving notices; the sender of the notice is obliged to
attach the notice in a visible position in his office, and the senior marriage Registrar is
expected to convey the notice to each marriage Registrar in the same district for
affixation.
o Sec. 17 deals with certificate issuance; the ministry must give a certificate to the parties
showing the issuing of the notice and issuing of the Declaration.
o Sec. 18 deals with the Declaration; if both parties are minors, they must appear in person
before the Minister and declare that they have no desire to marry and that all necessary
consents have been received as required by law.
o Sec. 19 deals with the approval of the father, guardian, or mother for marriage. If the
father is deceased, the consent of the guardian or mother is necessary. Minor Mother
permission is necessary for marriage if there is no Guardian.
o SECTION 20 deals with the right to prevent the issuance of a certificate by writing
notification to the Minister.
o SECTION 21 deals with the process after receiving notice. the notification in which the
Minister refuses to provide a certificate unless the ban of marriage is fulfilled following
examination

9|Page
o SECTION 23 deals with the issuance of a certificate to an Indian Christian. Before
granting the certificate, the Minister must determine whether the Indian Christian
understands the purpose and consequences of the notice/certificate.
o Sec. 24 deals with certificate form; the Legislation requires the certificate granted under
Sec. 17 to be in the form specified in the ACT 's second schedule.
o Sec. 25 deals with the solemnization of marriage once the certificate is issued. In the
presence of two witnesses, the minister has the authority to solemnize the marriage of
the parties.
o Sec. 26 deals with marriages that are not solemnized within two months after the date of
the certificate's insurance. If the parties do not get a new certificate, the certificate will
become null and void, and the marriage will be null and void.

6. A short note on Family Courts ACT


The Indian judiciary has the world's greatest backlog of outstanding litigation. According to the
National Judicial Data Grid (NJDG), around 3.7 million cases are waiting before India's High
Courts, District and Taluka Courts, or even more than 3.7 million for more than ten years. This
figure exemplifies the adage "justice delayed is justice denied." The normal courts are so
overburdened with civil problems that little attention was paid to family-related conflicts.
Several humanitarian organizations and people also placed pressure on the government to create
special courts to provide quick resolution to family-related conflicts. In its 59th Report (1974),
the LAW Commission emphasized the need of differentiating family-related conflicts from
regular civil processes and that legislative efforts should be undertaken to address family
disputes. As a result, the Family Courts ACT, 1984 was adopted by parliament in order to give a
quick resolution with fewer fees and formalities in issues connected to marriage and family, as
well as to create an understanding between the participants for their conciliation. This legislation
established Family Courts in the states, where reasonable attempts for a settlement are made
before initiating a trial in other Courts.
On September 14, 1984, the Family Courts ACT of 1984 was passed. This ACT is divided into
six chapters and 23 parts. The legislation was enacted with the primary goal of establishing
family courts for the quick and safe resolution of conflicts occurring in family and marriage and
related affairs. SECTION 3 of this ACT states that the State government, after consulting with
the High Court, must establish the Family Court in every region of the state with a demographic
of more than one million people or in any other location where the State government deems it
essential.
After consulting with the High Court, the State government should designate the extent to which
the Family Court's authority extends. It may also decrease, enhance, or change the Family
Court's jurisdictional limitations. SECTION 4 of the Family Courts ACT of 1984 deals with the
rules governing the appointment of judges in the Family Court. After consultation with the High
Court, the state government has the authority to nominate one or more judges to the Family
Court. After conferring with the High Court, the state government may select any of the judges
as the Principal Judge and any other judge as the Additional Principal Judge. The Principal
10 | P a g e
Judge's principal role is to allocate the court's work among the several judges, and the Additional
Principal Judge is appointed to execute the duties of the Principal Judge in his absences or when
he is unable to do so due to sickness or any other reason. This SECTION also outlined the
credentials necessary for appointment as a judge of the Family Court: He must have served in a
Judicial Office in India for a period of not less than seven years, or in the office of a Member of a
Tribunal, or any other job under the Centre or a State that needs particular knowledge of law; or
He must have served as an advocate of a High Court or two or more courts of inheritance for a
period of not less than seven years; or He must possess such qualifications as required by the
Central government after consultation with the Chief Justice of India; or He must possess such
requirements During the selection process for judges, it must be assured that the person chosen
knows how to resolve a conflict via mediation and counselling, to safeguard the marriage, and to
support the children's welfare through their reason and experience. When picking the judges, it
must also be assured that women are given precedence. The State Government will decide on the
salary or honorarium, any allowances payable, and other terms of service for Family Court
justices after conferring with the High Court.
SECTION 7 of this legislation grants the family courts the same powers and jurisdiction as the
District Court or Subordinate Civil Courts in their ACT ions and procedures. This SECTION's
explanation describes the nature of the lawsuits and processes, which are as follows: A litigation
or procedure for the declaration of dissolution of marriage, return of marital rights, or dissolution
of the parties' marriage; A litigation or procedure to determine the legitimacy of a person's
marriage or married status; A dispute or process involving the property rights of the parties to a
marriage; A litigation or process brought to obtain an injunction or order stemming from a
marriage; A litigation or procedure to determine a person's legitimacy; A maintenance claim or
process; A litigation or procedure seeking the person's guardianship or custody of a child.
SECTION 7(2) gives the family courts the authority to exercise the same jurisdiction as a
Magistrate of the First Class under Chapter IX of the Code of Criminal Procedure, 1973, as well
as any other jurisdiction allowed by law.
The family court process is extremely amicable, since no lengthy procedures are necessary. The
fundamental reason for the formation of the family court was to bring quick relief to the parties
via settlement. The main method followed by the family courts is outlined in SECTION 10 of
the Family Courts ACT of 1984. SECTION 10(1) applies the provisions of the Code of Civil
Procedure, 1908, to family court ACT ions or procedures, and by doing so, the family court is
regarded to be a civil court and has the authorities of such a court. SECTION 10(2) states that
the provisions of the Code of Civil Procedure, 1908, are applied to family court litigation and
procedures under Chapter IX of the code. SECTION 10(3) empowers the family court to
establish its own method based on the facts of the ACT ion or proceeding or the veracity of the
facts provided by one party and denied by another in order to reach a settlement. The hearings of
the family court might take place in front of cameras. According to SECTION 11 of the
legislation, the family court procedures may be conducted in private if the judge deems it
necessary or if any party to the complaint requests it. Because family courts operate with less
formalities, they do not record extensive testimony from witnesses; instead, just testimony
pertaining to the subject matter is recorded. SECTION 14 of the statute states that any report,

11 | P a g e
statement, or document relevant to the subject is admissible under the Indian Evidence ACT of
1872. Also, according to SECTION 15 of the ACT, it is not essential for a family court to
record the testimony of a witness in its entirety; just that portion that is relevant to the ACT ion
or procedure is required, and it must be certified by the judge and the witness.

7. Write a Note on Sapinda and Prohibited relationships

Sapinda connections are prolonged ties that span generations, such as father, grandpa, and so on.
Sapinda refers to a person who is linked by the same particles of the body, or it refers to a person
who is linked by the same pinda. The Hindu Marriage ACT of 1955 changed the notion of
Mitakshara and made marriage between Sapinda couples illegal. Unless such marriage or union
is allowed by LAW or by lawful tradition or usage. According to SECTION 3 of the ACT , the
custom must be legal. It was decided in the case of Harihar Prasad V. Balmiki Prasad that a
legitimate custom must be proven by clear and precise evidence; it is only by such evidence that
the courts can be convinced of their existence, and they must possess the requirement of
antiquity and clarity for legal recognition. Sapinda marriages are punishable under SECTION
18(b) of the ACT by simple imprisonment for up to one month, a fine of up to one thousand
rupees, or both.
Provisions Concerning the Sapinda-Sapinda Relationship
According to SECTION 3 (f)(i) of the Hindu Marriage ACT , 1955, a 'sapinda connection' with
regard to any individual goes as far as the third generation (inclusive) in the line of ascent via
mother, and the fifth generation (included) in the line of ascent through father, the line being
followed upwards in each instance from the individual concerned, who is to be considered as the
first generation; Two people are considered to be "sapindas" of each other if one is the other's
lineal ascendancy within the limitations of the sapinda relationship, or if they have a shared
lineal ascendant who is well within the boundaries of the sapinda connection with regard to each
of them.
The sapinda connection may be traced back to the third generation (included) in the mother's line
of descent and to the fifth generation (inclusive) in the father's line of descent. Sapinda
partnerships may span three generations for females and five generations for men. In Hindu
marriage, there are certain ties in which the wedding cannot be solemnized; these types of
relationships are known as degrees of forbidden relationship. The major goal of this LAW is to
avoid incestuous marriages, which are weddings between persons who have forbidden
relationships, such as siblings and sisters, children and grandchildren, and so on. The parties to a
marriage are not within the degrees of banned connection, according to SECTION 5(iv) of the
Hindu Marriage ACT, 1955, unless the custom or usage governing each of them approves of a
marriage between the two. This condition makes it plain that the people who are getting wedded
must not be in any form of banned connection unless the tradition or usage regulating them
allows it. Marriage formed within the degrees of banned connection will only become legal and
lawful if a valid custom exists. If a marriage falls into one of the degrees of forbidden
relationships, it is invalid by SECTION 11 of the ACT and punishable by simple imprisonment

12 | P a g e
for up to one month, a fine, or both under SECTION 18(b) of the same ACT. In Kamani Devi
V. Kameshwar Singh, it was held that even though the marriage was declared unlawful because
it was within the banned degree, the responsibility to support the wife remained.
Provisions Concerning Prohibited Relationship Degrees

According to SECTION 3(g) of the Hindu Marriage ACT, 1955, persons in prohibited
relationships are:
I. if one is a lineal descendant of the other; or
II. if one was the wife or husband of a lineal descendant or descendant of the other; or
III. if one was the wife of the other's brother, father's or mother's brother, grandfather's or
grandmother's brother.
IV. if the two are siblings or sisters, uncles and nieces, aunts and nephews, or offspring of
brothers or sisters, or two brothers or two sisters.
The connection also includes:
 A relationship based on half or uterine blood as well as a full blood relationship.
 Both illegitimate and genuine blood relationships
 Adoption as well as blood relationship
Following the passage of the Hindu Marriage ACT in 1955, important reforms were made in
Hindu law, removing the disparity between these two interpretations and prohibiting weddings
between people in these connections. There were also some additional degrees of connections
added to it.

8. Write a Note on Khula and Mubaarat

Khula and Mubarat are two kinds of mutual agreement divorce, although in either case, the
woman must leave with her dower or a portion of some other assets. "And it is not permitted for
you to take from women out of that whatsoever you have given them: unless (in the instance)
when both believe that they're not able to maintain within the bounds (set by Allah), in that case
there is no fault for either of them if the lady pledges herself," says the Holy Quran. In its
original connotation, the term Khula means "to pull," "to dig up," or "to take off," as in stripping
off one's clothing or garments. It is claimed that couples are like clothing to one other, and when
they take Khula, each puts off his or her garments, i.e., they get rid of one another.
In law, it is believed to represent an understanding between the spouses for the dissolution of a
conjugal partnership in exchange for compensation given by the woman to her husband out of
her assets. Whilst consideration for Khula is required, the official launch of the dower or transfer
of the property comprising the consideration is not a prerequisite for the khula's legitimacy. The
husband's permission results in an irreversible divorce. The husband has no authority to

13 | P a g e
terminate the 'Khula' since the consideration has not been provided. The consideration might be
anything, but it is generally mahr, in whole or in part. However, it might be any quality, as long
as it is not illusory. The most notable element of Mubarat is that both partners want to divorce.
As a result, the suggestion may come from either party. In Mubarat, both the husband and wife
are eager to get rid of each other. When the parties to a marriage engage into a Mubarat, all
reciprocal rights and duties are terminated.
However, Shia LAW is strict. It demands that both partners really find the married connection
annoying and inconvenient. The Sunnis have no set form, but the Shias insist on one. The Shias
require that the term Mubarat be accompanied by the word talaaq, or else there would be no
divorce. They also insist on the declaration being in Arabic unless the parties are unable to do so.
The desire to terminate the marriage should be stated unequivocally. Mubarat is binding on both
Shias and Sunnis. Other conditions are the same as in Khula, and the woman must go through the
iddat period. In all cases, divorce is largely an ACT of the parties, with no interference from the
court.

9. Write a Note on Iddat

In Islamic Law, the notion of Iddat is highly important. Iddat is the time that a Muslim woman
must follow after the breakdown of her marriage. During this time, a woman must stay in
seclusion and refrain from marrying another person; alternatively, a woman is barred from
marrying again following the dissolution of her first marriage. The goals of Iddat are twofold:
first, to determine the pregnancy of the wife in order to prevent misunderstanding about
parentage (child paternity), and second, to give both the pair time after Talaq to rectify or re-
think their relationship and decide whether to dissipate the marriage or rescind the Talaq
declared. Talaq will become irreversible after Iddat is completed.
The length of Iddat varies depending on the situation. In the particular instance of widowhood, a
woman is not permitted to marry before the finalization of 4 months and 10 days, and if she is
pregnant, her Iddat will be completed on delivery and miscarriage, but if it occurs before the
finalization of 4 months and 10 days, the remaining period she must perceive after delivery or
miscarriage. If a woman is menstruating, the time of Iddat in Talaq is divided into three courses;
otherwise, it is divided into three lunar months. If a woman's husband dies, she is required to
follow the Iddat period; however, if the marriage is not consummated, she is not required to
observe the Iddat period in the event of divorce.
According to Quranic verses, when a marriage is dissolved by divorce, the woman has the right
to maintenance throughout the Iddat term. She has the right to delayed dower (due after
dissolution), and if prompt dower has not been paid, it is becoming instantly payable. The
conclusion of Iddat will dissolve all marital ties between the pair, and the woman will not be
eligible to inherit rights in her ex-property husbands in the event of divorce.

14 | P a g e
10. Explain the provisions of The Parsi Marriage and
Divorce ACT regarding a divorce.

The Parsi Marriage is also viewed as a contract, and its legitimacy is dependent on a religious
ritual known as Ashirwad. Ashirwad literally translates to "blessings." A prayer or heavenly
encouragement to the spouses to faithfully fulfil their marital vows is required. The current Parsi
Marriage and Divorce ACT was enacted in 1865. Since then, conditions have changed
dramatically, as have the attitudes and ideas of the Parsi community to some measure.
 Matrimonial ACT ions are addressed under Chapter IV of the ACT. SECTION 30 states
that if the consummation of a marriage is made impossible due to natural circumstances, the
marriage may be deemed invalid and invalid at the request of either party.
 Dissolution Suits: According to SECTION 31 of the ACT, if a husband or wife has been
continuously away from his or her wife or husband for the length of seven years and has not
been known of as being alive, during that time, by those individuals who would have
normally known of him or her had he or she been living, the union of such husband or wife
may be broken at the request of either party.
 Grounds: Any union may be terminated by an action brought by any individual on any one or
more of the following grounds: That the marriage was not consummated within one year of
its solemnization due to the defendant's wilful reluctance to do so. That the defendant was of
unsound mind at the time of the marriage and has remained so up to the date of the
complaint. Under this basis, the plaintiff must establish that the plaintiff.
o Was not aware of the reality at the time of the marriage.
o Filed the complaint within three years after the marriage date.
o That the defendant has been incurably insane for two years or more promptly before the
filing of the complaint, or has been suffered regularly or occasionally from a mental
condition of such kind and severity that the plaintiff cannot fairly be expected to survive
with the defendant.
o That the defendant was impregnated by someone other than the plaintiff at the time of the
marriage. However, divorce shall not be granted on this reason unless:
o the plaintiff was unaware of the claimed fact at the time of the marriage.
o The complaint was brought within two years after the marriage date.
o Marital intercourse did not occur after the plaintiff became aware of the fact.
o That the defendant has committed adultery, fornication, bigamy, rape, or other unnatural
ACT since the marriage. However, divorce will not be granted on this basis if the
complaint is submitted more than two years after the plaintiff became aware of the truth.
 Divorce by Mutual Agreement: The Amendment ACT of 1988 included SECTION 32-B,
which allows for divorce by mutual consent. It states that, subject to the provisions of the
ACT, both parties to a marriage may file a divorce suit jointly on the grounds that they have
been residing independently for a period of one year or more, that they have been unable to
live together, and that they have unanimously agreed that the marriage should be
disintegrated.

15 | P a g e
o A complaint under this clause may be brought only one year after the date of the
marriage.
o After hearing the parties and doing such investigations as it deems necessary to
ensure that neither party's assent to the action was acquired by force or fraud, the
court issues a decree pronouncing the marriage to be terminated with immediate
effect of the decree.

11. Critically discuss the grounds of Divorce Available to


Only wife under the Hindu Marriage ACT, 1955.

When two people are legally bound by the institution of marriage and that connection is removed
with their permission, this is referred to as dissolution. The court then considers the processes of
child custody, alimony, and property partition. For the purpose of dissolution, couples may
choose between "no-fault" and "fault" divorce.
 While fault divorces are founded on specific wrongdoing by one party, no-fault dissolution
occurs when parties do not seek divorce based on any form of misconduct.
 In response to the aforementioned concerns, the Indian Parliament passed the Hindu
Marriage ACT in 1955. It confers specific rights and obligations on both parties while also
defining norms and regulations pertaining to notions like as Guardianship, Succession, and
Adoption. While marriage is a type of personal and social union between the parties, it is
important to recognize that the persons bound by matrimony have their own separate lives,
which are occasionally jeopardized as a result of this.
 When one or both of the parties' efforts fail to comply with their responsibilities under
marriage, the legislation enables the parties the option to dissolve the wedlock, provided the
criteria meet those specified or described in the ACT. These requirements are mutually
agreed upon by both parties and are offered to the wife on distinct grounds. This project
focuses on the rights granted to the woman in the event of divorce.
 If both parties' understanding fails, the marriage must be dissolved since it no longer
functions as a vital institution needed by the society. In such circumstances, the relationship
becomes a living hell for both sides, and that is not what this institution desires.
 Furthermore, the partnership necessitates the mental will of both individuals as well as the
will to live together. If such aspect is lacking, it does not fulfil its goal of social or personal
benefit for the parties, as indicated in the case of K. Srinivas Rao V. D.A. Deepa, wherein
the court defined the "irretrievable collapse of marriage" thesis. According to the court, "if
parties are not willing because marriage involves human sentiments and emotions, and if they
have dried up, there is hardly any chance of them sprouting back to life on account of
synthetic reunion created by court decree," but it does not currently serve as a valid ground
for divorce under the Hindu Marriage ACT.
 The fundamental goal in these circumstances is to alleviate the sufferings of both parties.
Individualism's growing popularity has heightened such awareness. There are certain
measures that must be taken before a marriage may be dissolved. These are designed in such

16 | P a g e
a manner that if there is even the tiniest chance of saving the marriage, it should be preserved
in such instances.
 9th SECTION (Restitution of Conjugal Rights)
o Following the successful conclusion of a marriage under Hindu LAW (SECTION 9),
both the husband and wife get certain unique privileges known as conjugal rights. The
three purposes of marriage, according to Hindu philosophy, are justice, reproduction,
and pleasure.
o All of the goals must be met while the couples are married in order for the marriage
to be successful. According to Hindu Law, "each spouse is entitled to the company
and comfort of the other, and if any spouse leaves any spouse without any valid
cause, the latter may seek the court for the decision of recovery of conjugal rights."
o As a result, if a husband abandons his wife for no apparent cause, the woman may
petition an order of restoration of conjugal rights.
o According to Suman Singh V. Sanjay Singh, "where there is evidence proving that
the respondent-husband departed from appellant's company without any legitimate
excuse, the appellant is entitled to a judgment for restoration of conjugal rights."
o Thus, rather than proceeding immediately to divorce in such circumstances, the court
assures that if the legitimate marriage between two parties can be salvaged, it should
be done before such a requirement emerges that both of the parties cannot live
together longer.
 SECTION 10 (Judicial Separation)
o In the event of a difficult marriage, the court permits a form of last option that the
parties may use to salvage their marriage under SECTION 10. After that, the partners
are under no duty to live together or cohabit.
o The parties have a choice of seven reasons on which to file a petition for judicial
separation. The court defined judicial separation as "no duty for either party to reside
with the other" in the case of Jeet Singh V. the State of U.P. Marriage-related
mutual rights and duties are suspended. However, the judgment does not separate or
dissolve the marriage. It provides a chance for repentance and change.
o Though judicial separation may constitute a reason for divorce after a specified
amount of time, it is not required, and the parties are not obligated to use that remedy,
and the parties may live retaining their position as wife and husband for the rest of
their lives."
 13th SECTION (Divorce)
o Even after all of these stages, if the parties are no longer able to be bound by the
institution of marriage, the court permits them to be free. To do so, the parties must
meet the requirements outlined in SECTION 13.
o Voluntary sexual intercourse, brutality, and abandonment are examples, as are mental
or psychopathic illness and venereal disease. In order to get a divorce, the party
demanding it must establish one or more of these reasons
o SECTION 13(1)(b) of this ACT specifies the exceptional grounds on which the wife
may seek a divorce. These are the grounds:

17 | P a g e
 If the husband marries another woman while his original wife is still alive, the
wife might use this as a legal cause for divorce. The sole need in such a
scenario is that the first wife be alive at the time of the divorce petition. Direct
proof is not required to demonstrate that the first spouse was present at the
time of marriage.
 If the husband has committed rape, sodomy, or bestiality against the wife, she
may cite this as a legal cause for divorce. It is necessary to establish by proof
that there was penetration for sexual activity in attempt to validate. Other
types of sex that become criminal offenses include sodomy and bestiality.
 If the marriage occurred before the female reached the age of fifteen. This
basis does not examine whether or not the marriage has been consummated.
However, the woman must use this reason for repudiation before reaching the
age of eighteen years.
 Courts Dismissed the Petition
o In some instances, even if the petition is submitted by divorcing parties, the court may
dismiss the petition. These are the grounds: In a case if the parties fail to produce
evidence to the court that is required for the claim, they are unable to satisfy their
claim. The petitioner's contention is unsubstantiated.
o If it cannot be shown in court that the adultery was committed for genuine, then the
party neglects the fact that adultery occurred. If the court discovers that the claimant
ignored the kind of wedding that has been filed against.
o If it is discovered that the petition was submitted in conjunction with any of the
respondents. In such a circumstance, the court will dismiss the party/parties' plea for
dissolution of marriage.
 The Court's decision to dissolve the marriage
o If the court is pleased with the parties' application for dissolution of marriage, it will
issue an order dissolving the marriage. The marriage is declared to be dissolved on
the day when a particular court issues a decree on the same subject.
o If the dispute is resolved between the parties, the judgment specifies all of the details
of the settlement. Every matter dealt with is detailed in the judgment paper. A
marriage is deemed to also be finally broken on the same day that the court signs the
parties' divorce decree. The State issues the certificate, which serves as confirmation
that the marriage has been formally terminated.
o However, in certain situations, the courts have the authority to refuse the request.
These are the cases:
 If the court learns that the claimant has committed adultery, the court has the
authority to dismiss the parties.
 If the court finds that the claimant has taken an unreasonable amount of time
to prove his case or to file charges for the execution of the ACT ion against
the other party.
 If the party fails to provide a plausible justification for splitting or deserting
his or her spouse prior to the other party's ACT of adultery.

18 | P a g e
 In the event that one party causes suffering or purposeful negligence to the
other.

12. Explain the provisions of Hindu Marriage ACT 1955,


under which divorce can be claimed by both the spouses.

The Hindu Marriage ACT of 1955 established the clause relating to the notion of divorce.
Divorce is defined under the Hindu Marriage ACT as the dissolution of a marriage. For the sake
of society, marriage or married relationships must be encircled by every legal safety for the
reason stipulated by law. Divorce is only permissible when there is a compelling cause to do so;
otherwise, other options are available.
 Adultery
o In many nations, the notion of adultery is not regarded a crime. However,
according to the Hindu Marriage ACT, adultery is one of the most essential
grounds for obtaining divorce in a marital infraction. Adultery is defined as the
consenting and intentional intercourse of a married individual with another
member of the opposite sex, married or unmarried. Even if the relationship
between the husband and his second wife is regarded bigamy, the individual is
accountable for adultery.
o The Marriage Laws Amendment ACT of 1976 added the notion of adultery into
the Hindu Marriage ACT. Sadanand Ghose V. Swapna Ghose, In this instance,
the wife saw her husband with another female resting on the same bed, and the
neighbor verified that the husband had committed an infraction. In this case, the
wife is granted a divorce.
o In the case of Sachindranath Chatterjee Vs. Sm. Nilima Chatterjee, the
petitioner and defendant in this case were married. After wedding, the husband
leaves the woman in his hometown so she may finish her education and relocate
to another city for job. He came two or three times a month to see her. Later, he
discovered that his wife had committed adultery, i.e., had sexual relations with his
own nephew, watchman, and so on. The plaintiff petitions the court for divorce
based on adultery, and his petition is granted, and the marriage is dissolved.
 Cruelty
o Cruelty encompasses both mental and physical manifestations. When one spouse
abuses or injures the other spouse, this is referred to as physical cruelty. However,
the idea of mental cruelty was introduced since the spouse might be
psychologically tormented by the other spouse. Mental cruelty is defined as a lack
of compassion that has a negative impact on a person's health.
o In Balram Prajapati VS. Susheela Bai, the petitioner filed a divorce suit against
his wife based on mental abuse. He demonstrated that his wife's behaviour with
him and his family was violent and unpredictable, and that she often filed false

19 | P a g e
complaints against her husband. The petition is accepted by the court, and the
divorce is granted on the grounds of cruelty.
 Desertion
o Desertion is defined as the permanent abandoning of one spouse by the other
spouse for no justifiable cause and without his permission. In general, one party's
refusal to accept the commitments of marriage.
o In the case of Bipin Chander Jaisinghbhai Shah Vs. Prabhawati, in this
instance, the responder leaves the home with the goal of abandoning his wife.
Later, the wife contacts the court, but the defendant demonstrates that, although
leaving the home with the aim to desert, he attempted to return and was stopped
from doing so by the petitioner. The defendant cannot be held accountable for
desertion in this case.
 Conversion
o If one of the spouses changes his faith to another without the approval of the other
spouse, the other spouse may petition the court for divorce. Suresh Babu Vs.
Leela is a case in which in this situation, the spouse becomes a Muslim and
marries another lady. Leela, the wife, filed a petition and requested a divorce on
the grounds of converting without her permission and cruelty.
 Insanity
o When a person is insane, they are mentally ill. The responder must have been
incurably insane in order for insanity to be used as a reason for divorce. The
respondent has been due to a mental condition of such a kind and degree that the
petitioner cannot fairly supposed to survive with the respondent.
o In the case of Vinita Saxena Vs. Pankaj Pandit, in this instance, the petitioner
sought a divorce from the respondent based on the respondent's diagnosis of
Paranoid Schizophrenia, which is a mental condition. She learned about them
after her marriage. In this case, the court approves the divorce based on the
husband's insanity.
 Leprosy
o Leprosy is an infectious illness that affects the skin, mucous membranes,
neurological system, and other organs. It is spread from person to person. As a
result, it is regarded as a legitimate reason for divorce.
o In the case of Swarajya Lakshmi VS. G. G. Padma Rao, in this instance, the
husband filed a divorce petition on the basis of leprosy. With the expert's findings,
he stated that his wife had incurable leprosy. In this case, he is successful in
obtaining a divorce on the basis of leprosy.
 Venereal Illness
o According to this principle, if the condition is infectious and may be spread to the
other spouse, it might be regarded a legitimate basis for divorce.
 Renunciation
o It implies that if one of the couples decides to give up the world and follow God's
way, the other spouse may go to court and request a divorce. The party that

20 | P a g e
renounces the world is deemed civilly dead under this view. It is a common Hindu
practice and is recognized as a legitimate basis for divorce.
 Death Presumption
o In this scenario, the individual is believed to have died if the person's relatives or
friends have not heard from him or her in seven years, whether living or dead. It is
a viable cause for divorce, but the burden of evidence is on the party requesting
the divorce.
 Divorce with Mutual Consent is a concept.
o According to SECTION 13B, a person may submit a divorce petition with the
mutual permission of both parties. If the partners desire to terminate their
marriage by mutual agreement, they must wait one year from the date of
marriage. They must demonstrate that they have been living apart for at least a
year and are unable to live together.

13. " Right of pronouncing a triple talaq conferred by


religion on a Muslim Man has now been curtailed.” Critically
discuss.

In India, immediate Triple Talaq was recently declared unlawful in a court of law, and a
legislation was enacted to that effect. It is an irreversible and immediate divorce in which the
husband repeats the word "Talaq" three times and thereby dissolves the marriage. In this sort of
divorce, the woman has no right to challenge the rationale or purpose for the Talaq, nor does she
have the right to an explanation. As a result, it is regarded anti-equality and anti-justice. Women
are left with severed relationships and no answers. This sort of Talaq does not even address the
issue of upkeep.
 The Supreme Court must intervene to protect women's rights. The court clearly excluded the
constraints, made direct exposure to places of worship easier for women, and held that
fairness shall overcome in all situations. Unnecessary limitations on their entrance hamper
their privilege to worship as well.
 22 nations, including Pakistan and Iran, have either formally or tacitly banned Triple Talaq.
It is often assumed that religious minority in a nation are resistant to change, maybe because
they fear that changes in their personal laws would push their faith to the point where their
religious identity will be jeopardized. This is plainly seen in India, and it has always been
difficult to amend personal laws, particularly when it comes to minority religions.
 In India, however, religious sentiments play a large role in decision making, and the lack of a
Uniform Civil Code is one of the greatest drawbacks in striking out certain procedures that
have been a part of the system since time immemorial but nobody feels it is appropriate to
understand the motives or legal arguments behind that particular method, for example, Triple
Talaq.
 A five-judge panel of the Hon'ble Supreme Court, by a majority of 3:2, has struck aside the
custom of Talaq-ul-biddat in the recent verdict of Shayara Bano V. Union of India. Justice

21 | P a g e
Jagdish Khehar (then CJI), Justice Kurian Joseph, Justice R F Nariman, Justice U. Lalit, and
Justice S Abdul Nazeer comprised the Coram.
 The minority view, led by the then-Chief Justice and Justice S Abdul Nazeer, unanimously
recognized Talaq-ul-biddat as an important religious practice under Shariat Law's ARTICLE
25 and so decided that the court could not intervene. However, they agreed that the practice
is discriminatory against women, but the fact that it has been a part of the Hanafi School for
1400 years is something that the court cannot overlook, and thus, it is the legislature, not the
court, that can intervene in this by enacting appropriate laws in this regard.
 The court cannot infringe on the right guaranteed by ARTICLE 25. They agreed that Talaq-
ul-biddat is evil in theology, but contended that it is regarded as legitimate legislation
everywhere and that just because it is poor in theology, it cannot be struck down.
 To get from "all individuals have an equal right to freely exercise religion" to "ARTICLE 25
preserves personal laws" is to torture language into a shapeless mush. Personal LAW is a
branch of religion. Religion is protected by ARTICLE 25(1). As a result, ARTICLE 25(1)
safeguards personal laws. That line of reasoning, however, overlooks the fact that ARTICLE
25(1) protects a person's freedom to exercise her religion rather than religious norms, rules,
or institutions; in other words, it protects individual rights rather than religious standards,
rules, or institutions.
 It could be argued that a Muslim man could confront the Court and contend that by rejecting
him the alternative of instant Triple Talaq, his ARTICLE 25(1) right was being infringed;
nevertheless, such a case is conceptually different from conferring the "stature" of
fundamental human rights upon an overall network of rules, and this distinction is critical.
"We are confident that the practice of 'Talaq-e-biddat' must be regarded as an intrinsic part of the
religious denomination in issue — Sunnis of the Hanafi school." There is no need for us to
record anything else. We believe that the practice of 'Talaq-e-biddat' has been sanctioned and
approved by the religious group that performed it, and as such, there is no question that the
practice is a component of their personal law."
 By respectfully disagreeing with the then-CJI, Justice Kurian Joseph (majority opinion)
denied admitting that just because Talaq-ul-biddat is an age-old tradition, it may be
regarded an important religious practice of Muslim law. He said that since the ACT of
1937 proclaimed Shariat to be law, "what is believed to be terrible in the Holy Quran
cannot be good in Shariat, and, in that sense, what is wrong in theology is also bad in
law."
 Together, Justices R F Nariman and U Lalit concurred that "this kind of Talaq is plainly
arbitrary in the sense that the marital connection might be destroyed wantonly and
fancifully by a Muslim man without any effort at reconciliation to salvage it." They
further claimed that the practice violated ARTICLE 14 of the constitution since the
Muslim male has ultimate authority in this respect while the woman has no voice in the
matter. Under ARTICLE 13, the court also invalidated the relevant portion of
SECTION 2 of the 1937 ACT (1).

22 | P a g e
As a result, a majority view has ruled that immediate Triple Talaq is discriminatory, violates
women's dignity, violates the concept of gender equality, as well as gender equity and
constitutional decency. It was also urged that the legislature enact appropriate legislation in this
respect.
 Following the ruling in 2017, the NDA administration, in light of the gravity of the issue,
introduced in parliament the Muslim Women (Protection of Rights on Marriage) Bill, 2018,
which declared the declaration of Talaq as in Talaq-ul-biddat to be unlawful and illegal under
SECTION 3.
o The Bill provides penalty for Muslim males who fall under SECTION 3 of the Bill,
which may be up to three years in prison and a fine. In the statement of objects and
reasons attached to the Bill, LAW Minister Ravi Shankar Prasad states that this
legislation will "help in making sure the larger Constitutional objectives of gender
justice and gender equality of married Muslim women and assist subserve their
fundamental human rights of non-discrimination and empowerment."
o The ACT was passed via Lok Sabha but could not be passed through Rajya Sabha,
and the bill expired owing to the conclusion of the parliament session and, therefore,
the end of the government's term. Though the LAW tries to improve the lives of
Muslim women who have suffered as a result of the centuries-old tradition of Talaq-
u-biddat, the manner in which it seeks to improve them has received much criticism.
o Talaq-ul-biddat was a common prACT ice until recently, and individuals followed it
for a variety of reasons. In the existing situation, the state cannot hold this ACT ivity
legally accountable. Civil culpability, on the other hand, may be a temporary remedy,
while criminal liability would be detrimental to the situation.
o How can incarcerating the husband for divorcing his wife through Talaq-ul-biddat
benefit the woman who filed a complaint against it? A LAW must be adopted to
prohibit the practice of quick Triple Talaq, but given the demands of the community
as well as the current scenario, strict measures in marriage matters will not assist, but
a solution that may help improve this situation and strengthen family bonds would.

14. Discuss the provisions of the Hindu Adoption and


Maintenance ACT regarding claim of maintenance by wife,
parents and children.

 wife's maintenance
o After divorce, the woman must be given support until she remarries. The concept
behind this is to let her to maintain her lifestyle and luxury that prevailed during her
marriage, and she must pay for it until she remarries.
o There is no set minimum or maximum amount for maintenance; it is determined by
the court based on the husband's earning abilities. If the husband is well-off, the
maintenance will be considerable in order to match the affluent lifestyle that the

23 | P a g e
woman was used to during the marriage. If that is not the situation, it must be a
sufficient sum to pay all of her reasonable expenditures.
o SECTION 18(2) of the Hindu Adoption and Support ACT specifies when a woman
is entitled to maintenance. According to the SECTION, a woman may live apart
from her husband and yet be entitled to support in the following circumstances:
 The husband left his wife by leaving her without justification, without
obtaining her approval, and by purposefully rejecting her wishes.
 The woman has been subjected to brutality throughout her marriage and
believes that remaining with her husband puts her life in jeopardy.
 If your spouse has an incurable and infectious condition.
 The spouse has another wife or mistress in the same home, or he lives
somewhere with another wife or mistress.
 The spouse has changed to another faith or there are other valid reasons why
the wife should live apart.
o Maintenance may be paid monthly or in one big fee. Even if the woman has a source
of revenue and some assets but need financial assistance for certain obligations such
as medical bills. If such expenditures are incurred, the spouse is obligated to pay
maintenance.
o The Hon'ble Supreme Court ruled the same in the matter of Smt. Anita Thaukral V.
Shri Satbir Singh Tkukral. In the aforementioned example, the wife had some source
of revenue and a nice residence, but she couldn't produce enough money to meet her
medical expenditures. The court ruled that the wife will get one of her husband's debit
cards, with the understanding that she would only take a fair amount for her medical
needs. In order to financially support her, a wife must be maintained after a divorce.
There are, however, certain exceptions of course.
o According to SECTION 18(3) of the legislation, a woman is not entitled to
maintenance if she has committed adultery or has any other unlawful sexual contact
with anybody else. Also, if she ceases to be a Hindu and converts to a religion that
does not fit within the scope of Hinduism.
o In the case of Abbayolla M. Subba Reddy V. Padmamma, the defendant had two
live wives, the second wife was demanding support, a bigamous union is forbidden
under Hindu Laws, and the legitimacy of the defendant's marriage with his new wife
was called into doubt. The Andhra Pradesh High Court ruled that if a man has two
wives, the marriage with the second wife is invalid ab initio since Hindu LAW
prohibits bigamous marriage and the parties never genuinely become husband and
wife.
o As a result, the second wife will have no right to maintenance since the marriage is
null and invalid from the start.
 Support for widowed daughters-in-law
o After a divorce, a husband is obligated to provide maintenance to his wife. If the
husband is deceased, his father is obligated to give maintenance to his daughter-in-
law. The same is stated in SECTION 19 of the Hindu Adoption and Maintenance
ACT, but the father-in-LAW is only obligated to pay upkeep if:
24 | P a g e
 his daughter-in-LAW has no income sources;
 she has no estate to maintain herself on her own;
 and if she has some estate, it is inadequate to satisfy her basic costs. If she has
no assets of her own and none of her husband's, parents', or children's property
is bringing her any support.
o The second clause of SECTION 19 further provides that a father-in-LAW is not
obligated to pay maintenance if: he is unable to do so from any coparcenary in his
possession; the daughter-in-LAW has no stake in such property, and such
responsibility will stop if she marries again. Children's and elderly parents'
maintenance
o People who are unable to make a living for themselves owing to acceptable reasons
must be provided with maintenance in order to satisfy their fundamental needs.
Children and the elderly are examples of such individuals.
o According to SECTION 20 of the statute, a Hindu man or female is expected to
support their offspring, whether they are legitimate or illegitimate. As long as they are
minors, children have the right to maintenance from their parents. Even after reaching
the age of adulthood, an unmarried daughter is entitled to support until she marries.
The provision also states that parents who are elderly or have physical or mental
disabilities must be cared for if they are unable to care for themselves.
o In the purpose of this SECTION, a childless stepmother will also be regarded a
"parent." The High Court of Andhra Pradesh decided in the matter of Mst. Samu Bai
& Anr V. Shahji Magan Lal that: Maintenance to old and infirm parents shall only
be paid if the parents have no means to support themselves or are unable to sustain
themselves out of their own property or wages.
o As a result, we may conclude that if the elderly parents have sufficient resources to
support themselves, the responsibility of children to support them can be lessened.
 Dependents' maintenance
o Dependents of a dead person must be supported if they lack the ability to do so on
their own. Dependents are defined in SECTION 21 of the legislation, and they are
entitled to support under SECTION 22. A dependent is someone who is financially
dependent on their parents, sibling, or another relative.
o According to SECTION 21 of the legislation, dependents in the context of this ACT
relate to the following relations of the deceased:
 He is a father.
 She is a mother.
 A widowed woman who has not remarried.
 A minor son, grandson, or great-grandson whose father and grandpa have
died.
 If he has been unable to acquire maintenance from any other source.
 Daughter, granddaughter, or great-granddaughter of a dead father and grandpa
who is unmarried.
 If she has not been able to acquire support from any other source.

25 | P a g e
 A widowed daughter who has been unable to get support from her husband's
inheritance, children, or in-laws.
 Widowed daughter-in-LAW or widowed granddaughter-in-LAW who has
been unable to receive support from other sources.
 An illegitimate minor son or unmarried illegitimate daughter.
o SECTION 22 of the statute specifies that dependents of a dead Hindu must be
supported by the inheritance that the deceased's heirs acquired. When the dependents
are not left with a portion of the property or estate by will or succession, they are still
entitled to be supported by whomever inherits the estate. If numerous people have
taken over the deceased's property, each of them is obligated to support the
dependents.
o The amount of maintenance due will be shared among them based on the value of
their part of the property. If a dependant obtains a portion of the deceased's property,
they are not obligated to support other dependents. Others who have taken over the
property will still be required to support other dependents, but the dependant who
owns a portion will be exempt, and maintenance will now be provided from the
residual property.

15. What are the Sources and schools of Hindu Law?

India as a country is a melting pot of people from many faiths. As a consequence, when the issue
of governance comes up, such religious convictions cannot be disregarded. This begs the issue of
what may be deemed to have the power of LAW and what can be recognized as sources for
formulating laws that control individuals of specific faiths. This kind of assessment is required in
the creation of laws that influence people's everyday lives, particularly their personal concerns
such as marriage, divorce, inheritance, and so on.
The principle of dharma governs Hindu law. Dharma is not clearly defined in Hindu law, but it is
understood to signify the way a person spends their life from birth to death. In other terms,
dharma is the guiding force that contains principles for correct individual behavior. This was the
primary rule of the ancient world, and no one, not even the monarch, was above dharma.
Sources: There are several documents and scriptures that control Hindu law, the expansion of
which is a natural result of a long-standing religion. This was supported further by laws passed
during the British era, and thus Hindu LAW may be separated between ancient and
contemporary origins.
1. Sources from the past: Writings such as the shrutis and smritis, as well as different
Upanishads, are ancient sources of Hindu law.
o Shrutis: Shruti is a Sanskrit term that meaning "to hear." According to Hindu law,
there are four Shrutis or Vedas: the Rigveda, Samveda, Yajurveda, and
Atharveda. The educated Brahmin class were the ones who'd repeat the written
vedas to the populace in ancient times, and whatever they said was accorded high

26 | P a g e
value and was considered the LAW of the nation. These describe the individual's
responsibilities and how they must be carried out.
o Smritis: The term smriti means to retain and refer to the SECTIONs of the shrutis
that the sages did not explain in their original form but instead wrote down in a
language that they were acquainted with. Smritis are classified into two types:
dharmasastras and dharmasastras. The former includes regulations about moral
standards of behaviour, while the latter contains rules concerning government,
interpersonal relationships, economic issues, and so on.
o Digests and Commentaries: Third, between the 7th century and 1800 AD, a
number of writers wrote commentary on the smritis. These are also essential
components of Hindu Law's basis. These included in-depth discussions and
reconciliations of significant components of the smritis. Medhatithi, for example,
wrote Manubhasya on the Manusmriti, while Vigneshwara wrote the classic
commentary Mitakshara in the Yajnavalkya.
2. Customs: The most significant, and maybe the oldest, type of law-making are the
customs, rituals, and behaviors that people have followed from time immemorial and that
have been recognized as LAW throughout time. These traditions are referred to
collectively as customs. A custom must be continuous, followed by a significant number
of people, non-discriminatory, and not contradict public policy in order to be regarded a
genuine source of law. The LAW in India recognizes three categories of customs: local,
class, and family.

3. Sources from the Present: The presence of the British on Indian beaches resulted in
significant modifications in Hindu personal rules. They enacted a slew of laws that
ultimately matured into the contemporary sources of Hindu law, which include as
follows:
o Justice, Equity, and Good Will: When there was no proper legislation to resolve
problems, judges made judgements based on the notions of Justice, Equity, and
Good Consciousness. This idea was founded on the premise of fair play; it did not
have a uniform implementation since the rationale of one judge may vary from
that of another; still, this doctrine played an important part in modernizing Hindu
personal laws.
o Legislation: Legislation refers to legislative ACT ions such as the Child Marriage
Restraint ACT of 1929, the Hindu Succession ACT of 1956, the Hindu Marriage
ACT of 1955, and the Hindu Minority and Guardianship ACT of 1956. The laws
approved bind all other sources of law. When LAW is codified, it means that it
has been correctly drafted and must be recognized and obeyed by all those who
control it. Legislation is frequently referred to as the legislative body in current
usage.
o Precedents: Following the establishment of British authority, courts were
established, and a hierarchy was established. Precedent suggests that lower courts
must follow the judgment of higher courts; if a certain case has previously been
determined, it looks rational to follow the same result if the circumstances of the

27 | P a g e
case are analogous to the decided case. In today's world, the judgment of the
Supreme Court is binding on all other courts.
4. Schools: There are two main schools of Hindu law: the Mitakshara school and the
Dayabhaga school.
o Mitakshara School: Mitakshara school includes marriage restrictions, a division
of society into four classes, and rites and rituals to be performed during
pregnancy; all of these rites and rituals are considered holy. Marriage between a
shudra and a brahmin is forbidden by the Mitakshara school; inter-caste marriages
are only authorized inside the higher caste. This school is further subdivided as
follows:
 The Banaras School
 Mithila High School
 Bombay Public School
 Dravida High School
o Dayabhaga school: The Dayabhaga school claims to benefit from succession
rules governance. This school had an immediate effect in that it included many
more people to the list of heirs for succession, which was not indicated in
Mitakshara school. The logic and reason of the Dayabhaga school of thinking take
precedence above commandments and precedents. In this approach, it promotes
more pragmatic and realistic viewpoints.

16. Discuss Cruelty as grounds for divorce.

Marital problems include delicate human and psychological ties. It requires mutual trust, esteem,
tolerance, affection, and passion, as well as adequate play to allow for fair adjustments with the
partner. The alliance must also follow social norms. Matrimonial behaviour is now governed by
regulations that were written with such norms and the new social structure in view.
Divorce, in definition, depends on the legal dissolution of a marriage, which allows individuals
to depart his or her spouse and be freed of spousal duties, subject to certain limitations.
 The divorce provision was first established in the Hindu Marriage ACT of 1955. The
Hindu Marriage ACT defines divorce as the dissolution of a marriage. Marriage or
marital partnerships must be surrounded by every statutory safeguard for the reasons
provided by LAW for the benefit of society. Divorce is only legal when there is a strong
reason to do it; otherwise, alternative choices exist.
 Cruelty is described as aggressive conduct. A simple disagreement, petty obnoxious
conduct, or disagreements between couples, on the other extreme, do not constitute
cruelty since they are common in regular married life. Cruelty must be terrible and
serious. Grave aggression is not frequently coupled with actual assault. Though actual
assault is a critical part of cruelty, a pattern of ill-treatment or mental or physical agony
against either spouse would also be considered cruelty.

28 | P a g e
 The day-to-day circumstances of a married existence create uncertainty in the capacity of
the pair to live happily with each other. Although there is no complete definition of what
all variables might lead to cruelty violation, if we look at a case of marital abuse
occurring around us, we may determine specific criteria such as:
o The spouse was assaulted physically.
o Committing adultery or having extra-marital relations with the spouse's
knowledge, and even confessing it publicly.
o In cases when one of the spouses is falsely accused of adultery.
o A constant show of pain and rage, including yelling or mistreating at the spouse.
o Undermining and restricting the spouse's capacity to be an independent person, as
well as forcing the spouse to exist in a marital union in which the partner has no
option but to rely on the other.
o They are not discussing any incident or fact regarding an established sexually
transmitted disease while they are already wedded.
 Either spouse's behavior should be such that it fits under the definition of cruelty under
Matrimonial Law. The Court must evaluate all of the factors that have contributed to the
couple's wish to divorce. Essentially, the Court must investigate the reason for the
marriage's collapse. Cruelty is defined in SECTION 13(1) (ia) of the Hindu Marriage
ACT as one spouse's conduct toward the other that leads the latter to have a reasonable
concern that it is no longer secure for him or her to continue in the marital relationship
with the other.
 Looking back at the Hindu Marriage ACT of 1955, we could see that harshness was
never recognized as a basis for divorce and was only utilized in cases of judicial
separation. In this scenario, the injured party or petitioner must show that the cruelty is so
extreme or unpleasant that it makes it impossible for him or her to keep living with his or
her partner (the defender). Nevertheless, the Supreme Court confirmed this in the
landmark decision of Narayan Ganesh Dastane Vs. Sucheta Narayan Dastane in
1975.
 As a consequence, the ACT was amended to include cruelty as a basis for divorce, and a
legal definition of the word cruelty was established under this ACT in 1976. The Court
did, however, emphasize that courts should only determine the question on the grounds of
harshness solely on the subject material of the lawsuit. Except for the inclusion of two
sentences, "consistently or often," there was no distinction between reasons of abuse
leading to judicial separation and grounds of cruelty resulting in divorce after this
legislation's amendment.
 This addition gave significantly more weight to proving cruelty as a ground for divorce
than showing it as a justification for judicial separation. This reason was introduced to
SECTION 10(1) of the Hindu Marriage ACT of 1955, and the term "cruelty" now has its
own meaning.
 In February 2007, the Supreme Court held in the landmark ruling of Mayadevi Vs.
Jagdish Prasad that any kind of mental cruelty endured by any spouse, not only women,
might constitute grounds for divorce.

29 | P a g e
o In this case, the respondent filed for divorce as a result of his wife's repeated
cruelty, as indicated by the husband (respondent), who said that the wife did not
give meals to him and his kids and rather criticized the husband and his relatives.
As a consequence, if a guy is abused, he has the right to divorce.

17. Explain" Dower". Discuss the remedies of a Muslim wife


when dower is not paid to her.

Mehr (dower) in Muslim LAW relates to money or assets that the wife is expected to claim from
the husband in exchange for the marriage, nevertheless this consideration is not same as that in a
legal agreement.
 Dower is a dowry that is placed on the husband as a gesture of respect for the wife. The
principal objective of the dower is to secure for the wife's sustenance after the dissolution
of her marriage, so that she never becomes defenceless in the event of the husband's death
or the marriage being terminated by divorce.
 Marriage was a completely different institution in pre-Islamic Arabia than it is now.
Various sorts of sexual relationships occurred between men and women throughout this
time period. Women were regularly the targets of abuse. Men used to forsake their wives
after despoiling them. Men used to fail to offer any financial aid to their wives after they
left them since there was no such suitable legal structure surrounding marriage. The
Shighar kind of marriage was used in ancient times. A man would marry his daughter or
sister to another in return for the latter marrying his sister or daughter to the earlier in this
sort of marriage. In this sort of marriage, the brides would get no dower.
 Another sort of marriage was a beena marriage. The husband met the bride but did not
bring her home in this style of marriage; the wife was addressed to as Sadiqa, and a gift
called as Sadaq was given to the lady upon marriage. Sadaq is regarded to be Islam's
earliest sort of dower.
 In the Baal wedding, the concept of mahr was established. Mahr was a sort of gift or
reparation made to the wife's parents in the Baal form of marriage. Usually, the mahr was
controlled by the wife's parents or guardians. The conventional style of marriage, on the
other hand, was progressively phased away over time.
 Islam's expansion evolved into a new form of nikah. This sort of marriage stipulated that
if a man divorces his wife, he must send her away with mercy, and that the man could not
reclaim the goods that had previously been granted to the wife. This Islamic marital ritual
gave birth to the concept of the husband repaying the woman on marriage as a means of
help in her old age. Mahr is entirely the wife's property, thus according Islamic law.
 A lot of jurists have defined Mahr. According to Wilson, a mahr or dower was a form of
payment for the wife's obedience of a person. A dower, according to Mulla, is "a sum of
money or property that the woman is allowed to receive from the husband in
consideration of marriage." According to Ameer Ali, a dower is a sort of consideration
that pertains to the wife.

30 | P a g e
 In the case of Abdul Kadir V. Salima (1886), it was determined that dower under
Muslim LAW is a sum of money or property pledged by the husband to be paid or
supplied to the wife in consideration of marital relationship, and that even if not explicitly
discussed at the time of marriage, the wife still has the entitlement of dower.
 Every woman has a right to a dower on the institution of marriage under Muslim law. If
such a privilege is violated, the woman, like any other citizen, has many choices. A wife
or widow has the essential rights to claim the payment of dower under Islamic law:
 Refusal to cohabitate: If the marriage has not been consummated, the woman has the
right to refuse residence with her husband as long as the dower is not paid on time. If the
woman is a juvenile or insane, the guardian has the authority to refuse to transport her to
her husband's home until timely dower is paid. The husband is required to support the
wife while she is residing at her guardian's house.
o However, if the woman consummates after marriage, she loses her complete right
to demand on timely dower payment. This is owing to the husband's capacity to
file a claim for the restoration of conjugal rights. If the lady wishes to continue to
cohabitate with her husband, she is only eligible to a dower order conditional
payment.
o In the case of Rabia Khatoon V. Mukhtar Ahmed (1966), it was ruled that if
the ACT ion is brought after sexual intercourse with her consent, the proper order
to issue is a decree for restitution, conditioned on prompt dower payment.
o In the case of postponed dower, the repayment of dower is a dependent
occurrence. As a consequence, the issue that arises is whether the wife may refuse
the husband's conjugal visits. There has been considerable debate over this.
According to Abu Yusuf, a well-known jurist, a woman has the right to decline
residence if a delayed dower is not paid. Imam Mahmood, a famous Shia LAW
expert, argues that in the case of postponed dower, the wife cannot decline to
cohabit.
 Dower rights as a kind of debt: The dower is a debt, as per the Lords of the Privy
Council, and the widow, along with other lenders, is eligible to have it satisfied out of the
husband's fortune following his death. If the spouse is still alive, the lady may retrieve the
dower by suing him. In the event that the dower duty is not met, the widow may pursue
her claim by filing a lawsuit against the husband's heirs. Nevertheless, the heirs are only
held liable for the amount and share of the deceased husband's property that they receive.
o In the instance of Syed David Hussain v Farzand Hussain (1937), it was held
that a Shia Muslim stood security for the payment of the dower by his underage
son. Following his death, his estate was held responsible for his son's mahr, and
each successor was found accountable for a part of the wife's case is based on his
share of the deceased's income.
 Possession privileges in lieu of delinquent dower: Dower is a debt, and the wife, like all
the other creditors, has the right to have it satisfied from her husband's estate after his
death. Her right is no more than that of any other unprotected creditor, save that if she
properly obtains the position of the whole or a part of his estate to satisfy her claim with
the rents and issues resulting from, she is allowed to retain such position until it is

31 | P a g e
fulfilled. The power of retention does not confer control of the property on her. As a
consequence, she is unable to sever ties with the land. A widow's right to retain
possession of her husband's inheritance in lieu of a dower is a picture special purpose. It
is done by force in order to induce early payment of the dower, which is an unsecured
debt.
 The consequences of apostasy on dower: Apostasy has a substantial impact on
Traditional personal law. It is said that a man's departure from Islam leads in a speedy
divorce from his wife. A woman's apostasy from Islam, on the other hand, doesn't really
end in the marriage's immediate dissolution. According to SECTION 5 of the
Dissolution of Muslim Marriage ACT 1939, a married Muslim woman has the same
dower rights as a married Muslim man upon the dissolution of marriage under this ACT.
The ACT 's dissolution of marriage, even if carried out after the wife's defection, does
not eliminate her of her title to dower.
 Appropriate for dower and restriction: If the widow's dower is not granted to her while
she is still alive, her heirs may be able to claim it after her death. The limitation period for
a lawsuit to collect prompt dower is three years from the date the dower is requested or
refused, per the Limitation ACT of 1963. The limitation period for delayed lien is three
years from the day the marriage ends due to death or divorce.

18. What is the difference between Judicial Separation and


Divorce?

Marriage is regarded as a sacrament in Indian society. It is an unbreakable bond between


husband and wife that is developed via rituals and customs. Prior to 1955, neither partner had
any recourse in the event of a broken marriage. They had no choice but to stay married and could
not divorce. Following the introduction of the Hindu Marriage ACT in 1955, things began to
alter in favour of both parties to the marriage. In the event of a failing marriage, the parties no
longer have to suffer in the marriage and can quickly end their marital relationship by Judicial
Separation or a Divorce judgment.
The Marriage Laws (Amendment) ACT of 1976 makes judicial separation and divorce more
prevalent. It is up to the parties to decide which of the two ways of dissolution to choose.
However, the legal consequences of judicial separation and divorce differ. A divorce is the last
nail in the coffin of a marriage, whereas a judicial separation allows for settlement between the
parties.
 Judicial Separation:
Either partner to a marriage, whether solemnized before or after the commencement of the Hindu
Marriage ACT, 1955, may submit a petition for judicial separation under SECTION 10 of the
ACT. After a decision is issued in favour of the parties, they are not required to cohabit.
However, some matrimonial rights and obligations continue to exist. During the term of
separation, they are not permitted to remarry.

32 | P a g e
They are free to live apart from one another. During the period of separation, rights and
responsibilities are suspended. The reasons for judicial separation are the same as the grounds for
divorce. Judicial separation may be requested under SECTION 13(1) for the following reasons:
i. Adultery: If the other spouse had a consenting sexual relationship with someone
other than his or her spouse after the marriage was solemnized.
ii. Cruelty: When one of the spouses treats the other cruelly after the marriage is
solemnized.
iii. Desertion: If the other party has abandoned the spouse for a continuous period of two
years for no justifiable cause immediately preceding the filing of the petition.
iv. Conversion: If one of the spouses is no longer a Hindu.
v. Insanity: If the other party is insane or has been suffering from a mental disease of
such a nature and extent that the petitioner cannot live with the other party.
vi. Leprosy: If the other person has had a severe and incurable type of leprosy.
vii. Venereal disease: If the other person has an infectious kind of venereal disease.
viii. If the other spouse has forsaken the world by joining a religious order, he or she
has renounced the world.
ix. For the past seven years, he has not been heard alive.
In addition to these grounds, others are specifically dedicated for women:
i. Husband has more than one wife living: If the husband married before the ACT 's
inception and remarried after the ACT 's inception, any of the wives can file a judicial
separation petition as long as the other woman is alive at the time the petition is filed.
ii. Rape, Sodomy, or Bestiality: If a man is found guilty of rape, sodomy, or bestiality,
his wife may file a petition for judicial separation.
iii. Marriage before the age of fifteen: If a woman's marriage was solemnized before
the age of fifteen, she might reject it after the age of fifteen but before the age of
eighteen.
In the event of a judicial separation, the court can also decide on issues such as wife's support,
custody of children, and property
In the case of Sohan Lal Vs. Kamlesh, it was decided that after a judicial separation, a wife is
entitled to maintenance from her husband if she is unable to support herself.
What should be done if the parties desire to resume cohabitation after a court separation
Because a judicial separation decree is a decision in rem, if the parties desire to resume
cohabitation, they must get the order of judicial separation cancelled by the court. Normally, the
court revokes the degree upon the presentation of the petition with the approval of both parties.
Judicial Separation is a first step to divorce. The goal of judicial separation is to give the parties a
chance to work out their differences.
Divorce:

33 | P a g e
When a couple divorces, they no longer have the status of husband and wife. Divorce ends the
marriage and terminates all mutual rights and duties. The parties are allowed to remarry.
SECTION 13 discusses the reasons for divorce. Divorce and judicial separation have the same
reasons. Aside from these grounds, the woman may seek divorce on the other grounds mentioned
above.
The parties are also able to file a petition if there is no return of cohabitation between the parties
to the marriage for a year or longer after the court orders judicial separation. In such a
circumstance, the court will not seek proof of any of the divorce reasons. The court will award a
divorce decree based only on the presentation of the petition.
In this instance, the court has ordered the return of conjugal rights under SECTION 9 of the
HMA, 1955, and the parties have refused to comply with the court's ruling and have failed to
cohabit. In such a circumstance, upon presentation of the divorce petition, the court will not
inquire into any reasons for divorce and will issue a divorce judgment based on the failure of
restitution of marital rights.
Though the petitioner cannot substantiate grounds for divorce or the court is not convinced that
the ACT is bad enough to warrant a divorce judgment, it has the authority to issue a decree of
judicial separation even if the petitioner did not request it. In the case of Vimlesh V. Prakash
Chandra Sharma, the court ruled that a single ACT of cruelty is not serious enough to warrant
a divorce decision. As a result, the court issued a judicial separation decision to give the parties a
chance to reconcile.
SECTION 13(b) of the Marriage LAW (Amendment) ACT of 1976 adds a new cause for
divorce. When both parties believe that their marriage is irreparably broken and that there is no
hope of reconciliation, they may present a divorce decree by mutual consent under SECTION
13(b), in which case the court will not inquire as to the reason for the divorce and will grant a
decree in favour of the parties if both of them want a divorce. On the introduction of a petition
for divorce by mutual consent, the ACT grants a 6-month term for reconciliation.
However, in the case of Nikhil Kumar V. Rupali Kumar, the Supreme Court abolished the six-
month required reconciliation period. Divorce on the basis of mutual consent can now be granted
on the presentation of the petition, and the parties do not have to wait six months.

Judicial Separation Divorce


A petition can be filed at any moment after Only after one year of marriage may you
the marriage. file.

There is just one step of judgment. If the The process of judging consists of two
criteria are met, the decree is granted. steps. First comes reconciliation, then
comes divorce.

Marriage is temporarily halted. Brings an end to a marriage.

34 | P a g e
Cannot remarry after the decree's enactment Can remarry once a divorce order is issued.
It is a valid reason for divorce. -

Judicial Sep. is satisfied with a single It is important to be in an adulterous


incident of adultery. relationship.

The prospect of reconciliation. There is no chance of reconciliation.

19. Write a Note on Nullity of Marriage

Marriage nullity is a court pronouncement that marriage did not occur. It is a legal term that
refers to the legality of marriage. It signifies that the parties did not have a lawful marriage
conducted
Under Hindu Law:
According to smrities, marriage is a necessary sanskar for Hindus. It is one's responsibility to do
so. Marriage was inseparable and necessary for carrying out religious and spiritual
responsibilities. Prior to the parliamentary passage, there was no idea of marriage termination or
nullity in Hindu personal law, and marriage was viewed as a holy and powerful wedlock for life.
However, with the implementation of the Hindu Marriage ACT of 1955, there are some reasons
for declaring a marriage null and invalid. These justifications are specified in Clauses I (iv), and
(v) of SECTION 5 of the Hindu Marriage ACT of 1955. The following are the grounds:
i. If either side had a surviving spouse at the time of marriage, this is referred to as
bigamy.
ii. If a banned degree relation marries, unless conventions and usage allow it,
iii. If a sapinda marries another sapinda, unless conventions and usage permit such a
marriage, the sapinda marries the sapinda
The Hindu Marriage ACT of 1955 recognizes sagotra marriage.
There are also voidable marriages, which are legitimate until they are ruled null and void. A
voidable marriage shall be declared null and void by a declaration of nullity issued under
SECTION 12 of the Hindu Marriage ACT, 1955. The parties might choose to continue their
marriage or have it annulled by a judicial order. The following are the reasons
i. Respondent's impotency
ii. Incapacity to offer valid consent or compelled consent of parties, mental disease,
or person unsuited for child reproduction
iii. Marriage at a young age
iv. If the responder was pregnant by someone else at the time of the marriage.

35 | P a g e
Nullity under Muslim personal law:

Marriage is a dissolvable contract in Islam, but it is indissoluble in Hinduism. Marriage is


considered as a contract in Muslim personal law, requiring the lawful consent of both parties and
the determination of ‘Mehar.' As a result, divorce is legal in both the Shia and Sunni sects.
Marriage is invalid under the Dissolution of Muslim Marriage ACT of 1939 and personal LAW
if the parties or their guardians do not give legal permission. There are various more reasons for
declaring a marriage null and invalid. The following are the grounds:
i. Woman's interreligious marriage has no religious validity. A Muslim guy cannot
also marry a woman who does not believe in Islam.
ii. Marriage between milk relatives, also known as 'maharim,' or near blood
relatives.
iii. Marriage with a person who has renounced Islam or who does not believe in the
principles of Islam.
iv. Sunni LAW forbids temporary or conditional marriage.
v. Marriage to a lady during the iddat era.
vi. Where marital conditions violate Islamic principles.
Nullity under Christian law:

Marriage's standing has altered as Christianity has evolved. In Christianity, matrimony is also
indissoluble and sacred, and it is a public religious ritual. As a result, nullity of marriage is
difficult to obtain. However, in order to advance society and eliminate prejudice against Indian
Christians, distinct marital LAW has been enacted. The Indian Christian Marriage ACT, 1872,
was enacted, as was the Indian Divorce ACT, 1869, for their divorce or nullity of marriage. In
the year 2001, this ACT was changed. Marriage can be ruled null and invalid on the following
grounds, according to this ACT:
i. Respondent was impotent at the time of marriage and at the time of the
establishment of the suit. Either party had a live husband or wife at the time of
marriage and that marriage is still in existence, i.e., bigamy.
ii. Marriage between people who are related to each other in a forbidden degree of
consanguinity or affinity
iii. At the moment of marriage, either partner was insane or an idiot.
Consent is not a reason for nullity of marriage under the Indian Divorce ACT of 1896.
Nullity under Parsi Marriage and Divorce ACT of 1936.
In India, the Parsi group has their own marital law. Under SECTION 30 of this ACT, if
consummation of marriage is impossible owing to natural causes, the marriage might be declared
null and invalid at the request of the parties.
Marriage is declared null and void under the Special Marriage ACT of 1954.

36 | P a g e
According to SECTION 24 of the ACT, a marriage can be declared null and void by a
declaration of nullity on the following grounds
i. Neither side has a spouse who is still alive.
ii. Incapable of giving legitimate permission owing to incapacity of mind or mental
disease, or unsuited for childbearing
iii. The parties are underage.
iv. Parties are in a forbidden degree relationship.
v. Respondent impotency
There are various more reasons for declaring a voidable marriage null and void
i. Marriage has not been completed owing to respondent's intentional unwillingness.
ii. If the responder was pregnant by someone else at the time of the marriage.
iii. According to the Indian Contract ACT of 1872, either party's consent was gained
by deception or coercion.
The Special Marriage ACT grants legal validity and protection to interreligious marriages
performed in accordance with its stipulations. Under this ACT, any person, regardless of caste or
religion, may marry.
The procedure for getting a decree of nullity of marriage is essentially the same in every personal
law. A petition for nullity of marriage must be filed with the court. The court's jurisdiction is
determined by where the defendant or respondent resides, where the marriage was solemnized,
or where the parties last resized together. The court will next issue a notice to the respondent or
defendant to appear in court and respond.
Following a hearing and evidence, the court grants the requested remedy. A court formed under
the ACT is referred to as a court under parsi law. According to the Hindu Marriage ACT of
1955 and the Special Marriage ACT of 1954, the court is either a family court or a municipal
civil court. Under Muslim law, matters are settled by Islamic practice rather than by courts.
When a declaration of nullity of marriage is made, the court also decides on the amount of
maintenance to be paid to the opposing party, which can be monthly, annually, or in a lump sum.
Children born from this marriage are considered genuine. Nullity of marriage is a statement that
no marriage existed and that the parties are no longer husband and wife. They are allowed to
marry anyone they choose. The declaration of nullity of marriage affirms that there was no status
of marriage between two people. Marriages that are declared null and invalid have no legal
standing. They are illegal and not legally enforceable.

20. Write a Note on Irretrievable breakdown of marriage


Modern society has grown highly complicated as a result of changes in socioeconomic situations,
the fragmentation of the united family structure, fast industrialization and urbanization,
education, and work. Furthermore, the laws that have granted women equal status and rights
have had a significant influence on the institution of marriage, which is no longer seen as an
irreversible partnership. All around the world, there has been significant legislative and judicial

37 | P a g e
intervention in the realm of marriage laws. Divorce, which was once seen as a sin, has codified
regulations that are being significantly amended and liberalized.
In India, the Government of India sought to incorporate 'Irretrievable Breakdown of Marriage' as
a reason for divorce in the Hindu Marriage ACT and Special Marriage ACT, in accordance with
the recommendations of the LAW Commission of India's 71st report.
Irretrievable Breakdown of Marriage is defined legally as
"A circumstance in which either or both couples are no longer able or willing to live with each
other, ruining their husband-and-wife connection and leaving no possibility of resumption of
spousal obligations."
In other terms, irretrievable breakdown of marriage can be described as such failure in the
married connection or such adverse conditions to that relationship that there is no reasonable
prospect of the couples continuing together as husband and wife for mutual comfort and support.
The LAW Commission of India's 71st report, published in 1978, addresses the notion of
Irretrievable Marriage Breakdown. The Report is based on the initial question of whether
Irretrievable Breakdown of Marriage may be added as a reason for divorce under the Hindu
Marriage ACT and under what conditions.
According to the Report from 1920, New Zealand was the first of the Commonwealth countries
to adopt the concept of a three-year separation agreement as a basis for filing for divorce in the
courts. The Court of New Zealand awarded the first divorce on the grounds of irretrievable
breakdown of marriage in 1921.
The Court ruled that "when marriage connections cease to exist, it is neither in the interests of
the parties nor in the public interest to maintain the man and woman linked as husband and wife
in law." The Court further stated that "in the case of such separation, the primary purpose of
marriage is thwarted, and its continued continuation is not only futile but malicious." This
resulted in the development of the breakdown hypothesis in matrimonial law.
The current view of divorce is represented by the breakdown theory of divorce, which is
inextricably linked with the no fault theory of divorce. According to this idea, the LAW
recognizes a scenario and tells the unhappy couple: if you can persuade the Court that your
marriage has broken down and that you want to end a condition that has become unbearable,
your marriage will be dissolved, regardless of the cause. When the marriage's objectives cannot
be met, the marriage is considered to have shattered. When there isn't even a glimmer of hope
that the partners may be restored, the marriage is regarded irretrievably broken.
Although the notion of irretrievable breakdown of marriage being designated a reason for
divorce under the Hindu Marriage ACT, 1955 has been much more contested, it has also been
criticized at different stages by state High courts and the Government of India.
The High Court criticizes: In numerous occasions, the High Court has voiced its dissatisfaction
with the notion that the Hindu Marriage ACT, 1955 be altered to make irretrievable breakdown
of marriage a good reason for granting a divorce decision. The judges of the High Courts have

38 | P a g e
spoken out against the inclusion of irretrievable breakdown as a divorce reason. One of the
reasons raised in the High Court's response is that it is quite difficult to claim that the husband
and wife will never live together simply because there has been a schism between them, and for
the time being, there appears to be no chance of their living together.
The mere fact that there has been a schism between the parties or that they are now living apart
does not imply that the marriage has ended. It is possible that what appears to one individual to
be irreversible may look to another to be not yet beyond repair. However, such a condition of
affairs cannot be allowed to continue indefinitely, and there must come a moment when one of
the parties is authorized to seek a court ruling on whether or not the marriage can be recovered.
The Government of India, Ministry of Education, Department of Social Welfare, has expressed
concern that making irretrievable breakdown of marriage a ground for granting a decree of
divorce is unnecessary in light of the fact that sufficient grounds covering 'irretrievable
breakdown of marriage' exist in the Hindu Marriage ACT and the Marriage Laws Amendment
ACT , 1976, for the purpose of seeking divorce
According to news reports dated February 19, 2015 and July 12, 2015, the current NDA
administration may repeal the Marriage LAW Amendment Bill 2013, which was introduced by
the previous UPA government. The Bill included SECTION 13C, Irretrievable Breakdown of
Marriage. The bill was passed by the Rajya Sabha on August 26, 2013; however, it could not be
debated in the Lok Sabha owing to a change in the government at the Centre.
Though the current government considered tabling the bill again, the then-LAW Minister, Mr
Sadanand Gowda, revealed that the government was still evaluating the ramifications of the Bill
after receiving more than 70 objections against it.
Despite the fact that the Bill was created to fill a gap in divorce law, opponents of the Marriage
Laws Amendment Bill argue that if implemented, the Bill will lead to an increase in illicit and
live-in relationships, eroding the institution of marriage and family values. Another concern that
the groups have is a rise in crime and excessive litigation
Given the current situation, it looks that the Marriage Laws Amendment Bill will not see the
light of day in the foreseeable future, despite the Hon'ble Supreme Court's repeated requests for
its inclusion.

21. Discuss the constitutional validity of SECTION 9 of


Hindu Marriage ACT, 1955 relating to Restitution of Conjugal
Rights.
When either the husband or the wife has withdrawn from the society of the other without
reasonable excuse, the aggrieved party may petition the district court for restitution of conjugal
rights, and the court, if satisfied of the truth of the statements made in such petition and that there
is no legal ground why the application should not be granted, may decree restitution of conjugal
rights accordingly.

39 | P a g e
SECTION 9 of the Hindu Marriage ACT (HMA), 1955 and SECTION 22 of the Special
Marriage ACT (SMA), 1954 provide for restitution of conjugal rights, which states that if a
husband or wife withdraws from the society of the other without any reasonable excuse, the
aggrieved party may file a petition before the District Court for restitution of conjugal rights, i.e.,
to bring the spouse back to live with the other spouse. If the court determines that the petition is
truthful and that there are no legal grounds for rejecting the petition, it may issue a decree of
conjugal rights.
However, it is the responsibility of the person who has withdrawn from the other's society to
demonstrate that there is a justifiable basis for his/her departure and therefore that the decree
should not be issued.
According to SECTION 9, the following are critical criteria for carrying out the restitution of
conjugal rights:
i. Withdrawal of a spouse from the other spouse's society for no apparent cause.
ii. The petitioner's statement has been accepted by the Court.
iii. There is no legal basis to dismiss the petition.
This SECTION has several ambiguities and words that are not properly defined.
Terms like 'withdrawn' and 'reasonable justification' have not been defined in this SECTION.
Even if the withdrawal was forced rather than voluntary, the burden of evidence falls on the
individual who withdrew from society, which may or may not have occurred freely. A fair
justification, similarly, is ambiguous in the sense that what is acceptable for one person may not
be reasonable for another.
Questions such as "what does withdraw from society mean?" and "Does a wife's unwillingness to
leave a job equivalent to withdrawal from society?" have been questioned in court several times.
In Tirath Kaur V. Kripal Singh, the woman left her husband's home to pursue her career. Her
spouse paid frequent visits, and she used to pay him a share of her earnings. When his financial
expectations escalated and the wife was unable to meet them, the husband requested her to resign
from her employment. When the wife refused to leave her employment, the husband petitioned
the Punjab & Haryana High Court under SECTION 9 of the HMA, 1955 to have her returned.
After relying on the Mulla's opinion that it is the wife's primary duty to submit herself obediently
to her husband's authority and to remain under the same roof with his protection, Justice Grover
stated that no rule or principle has been shown to him that will uphold that the wife could be
allowed to withdraw 'virtually' from the husband's society in this manner. As a result, the wife's
unwillingness to leave her employment equates to her seclusion from society. This line was then
used to base a series of decisions
However, in the case of Shanti Devi V. Ramesh Chandra Roukar and Ors., the Allahabad
High Court issued a contrary decision, holding that the wife's refusal to quit from her
employment did not constitute withdrawal from society. As a result, it cannot serve as a
sufficient basis for awarding a ruling under the restitution of marital rights.

40 | P a g e
Furthermore, in the case of Smt. Vibha Shrivastava V. Dinesh Kumar Shrivastava, the
Madhya Pradesh High Court held that a wife's refusal to leave her job does not constitute
withdrawal from her husband's society, and thus it is not a sufficient ground for the husband to
seek relief for restitution of conjugal rights.
SECTION 9 has long been a source of contention in terms of its constitutionality. The Andhra
Pradesh High Court, on the other hand, took the matter seriously in 1983. In T. Sareetha V. T.
Venkata Subbaiah, the High Court ruled that SECTION 9 of the HMA, 1955 was
unconstitutional because it violated the right to privacy and human dignity provided by
ARTICLE 21 of the Indian Constitution. According to Justice P.A. Choudhary, SECTION 9 is
the most egregious breach of the right to privacy. Forcing a spouse to have sexual intercourse
with her spouse takes away her right to govern her own body. The state cannot compel a spouse
to continue her free union with her spouse in their relationship. Coercion can neither ease a
couple's ruffled sentiments nor clear up their misunderstanding. The judge even stated that
SECTION 9 does not serve any public purpose and instead violates the right to equality
guaranteed by ARTICLE 14 of the Indian Constitution.
However, in the case of Harvinder Kaur V. Harmander Singh, the Delhi High Court revoked
the judgment, and the Supreme Court overruled it in the case of Saroj Rani V. Sudarshan,
where regard was given to family, togetherness, and sustaining marriage relations between
husband and wife.
The Supreme Court cited its previous decision in the case of Gobind V. State of Madhya
Pradesh, in which the court stated that "the right to privacy protects the personal intimacies of
the home, the family, marriage, motherhood, procreation, and child-rearing," which means that
there can be no legal interference within the private space of a home
Furthermore, in the case of Saroj Rani V. Sudarshan Kumar Chadha, Justice Mukherjee said
that the objective of SECTION 9 is to reconcile an estranged couple so that they can live
together in a marital residence in peace. On the issue of compelling a spouse to have sexual
relations with her spouse, Justice Mukherjee stated that the impugned SECTION's goal is
'consortium,' not sexual relations. The goal of issuing a decree under this SECTION is to
cohabitate the couple in the same household with love, care, and tenderness, and it in no way
obligates the parties to engage in sexual intercourse.
However, because marital rape is not a crime in India, forcing a wife to stay with her husband is
equal to taking away her option of having sexual intercourse or not, because the husband can
violate her right over her body at any moment without facing legal consequences.
ARTICLE 19 violation
The return of conjugal rights decree violates:
i. 19(1)(c) Freedom of Association
ii. (19)(1) (e)Freedom to reside and settle in any part of India
iii. 19(1)(g) Freedom to practice any profession

41 | P a g e
When an order of restitution of conjugal rights is granted, a spouse is forced to remain with his
or her spouse against his or her desire. This is a violation of the right of association provided by
ARTICLE 19 of the Indian Constitution. In Huhhram V. Misri Bai, the woman left her
marriage because her father-in-LAW had an evil eye on her and the husband mistreated her.
Even after that, the Madhya Pradesh High Court issued a reparation order to the spouse. It may
be concluded that if she was abused by her father-in-LAW as a result of her relationship with her
husband as a result of the decree, the court's decision would be held liable for the disaster.
Furthermore, in the case of Atma Ram V. Narbada Devi, the Rajasthan High Court issued a
restitution decision in favor of the wife notwithstanding the husband's unambiguous statement
that he does not wish to live with her. This certainly infringes the husband's right of association.
ARTICLE (19)(1)(e) grants the right to stay or establish in any area of India, whereas
ARTICLE (19)(1)(g) grants the right to prACT ice any profession. Because a spouse is obliged
to come and live with another spouse in his/her marriage house, the decree of restoration of
conjugal rights violates the freedom to reside in any region of India.
Furthermore, in many circumstances, it infringes the freedom to prACT ice any profession.
In the case of Tirath Kaur V. Kripal Singh, the woman was working and needed to be apart from
her husband. However, owing to various disputes, the husband requested that his wife leave her
employment, and when she refused, he filed a petition for recovery of conjugal rights. The ruling
was approved by the court, requiring the woman to reside with her husband. This is a blatant
infringement of the right to live and work in any area of India.
According to a study released in 2015 by the High-Level Committee on the Status of Women,
restoration of marital rights has no relevance in independent India. In its 'Consultation Paper on
Reform of Family Law' in 2018, the LAW Commission of India recommended the elimination
of SECTION 9 of the HMA, 1955, and SECTION 22 of the SMA, 1954, based on the report's
recommendations.
Furthermore, in K.S. Puttaswamy V. Union of India, the Supreme Court recognized the basic
right to privacy, which gives people entire sovereignty over their bodies.
Furthermore, in the case of Joseph Shine V. Union of India, the Supreme Court said that the right
to privacy is dependent on individuals exercising their autonomy. If an individual is unable to
exercise his or her right to privacy, the court must take efforts to ensure that the person's right is
fully realized. The Court further stated that treating familial structures as private space does not
violate an individual's right to privacy.
Following these judgments, a petition was filed in the Supreme Court contesting the
constitutionality of SECTIONs 9 and 22 of the HMA. The petitioner stated that giving an order
for recovery of conjugal rights constituted a "coercive ACT " by the state compelling a spouse to
live with another spouse against his or her choice. Furthermore, the conduct violates one's sexual
autonomy, right to privacy, and right to live a decent life, as protected by ARTICLE 21 of the
Indian Constitution.

42 | P a g e
The petitioner further stated that, while these parts grant both husband and wife the ability to
petition the court, they are discriminatory against women and treat women as 'chattel' under these
statutes.
The plea has been forwarded to a larger bench after being heard by a two-judge panel led by
former Chief Justice of India Ranjan Gogoi. The petition is being heard by a three-judge panel of
India's Supreme Court.

22. "The remedy of Divorce by mutual consent is recognized


under every personal law". Discuss.

When both parties agree to divorce in a polite manner, which is a more amicable method than
battling in court and defaming one other. In that circumstance, they may petition a District Court
under SECTION 13B of the Hindu Marriage ACT, 1955, and the Court may award them a
divorce decision. Divorce proceedings before the Panchayat will have no effect; the parties must
go via Matrimonial Courts.
Divorce by mutual consent is provided for under a number of personal laws, including the Hindu
Marriage ACT of 1955.
Divorce by Mutual Consent is provided for under SECTION 13B of the Hindu Marriage ACT
of 1955. It indicates that if the parties have been living separately for a year and are unable to
live together and have agreed to separate mutually, they can seek divorce by mutual agreement.
Personal Laws and the Muslim Women (Protection on Divorce) ACT of 1986
Muslims can seek divorce via mutual consent under the Muslim Women (Protection on Divorce)
ACT of 1986 and Personal Laws. In Muslim Personal Law, there are two forms of divorce by
mutual consent:
Mubarat Khulla
There is no need to give a reason for divorce under either Khula or Mubarat, and no
consideration flows from wife to husband under Mubarat. As a result, the woman (in the case of
Khula) or the husband and wife (in the case of Mubarat) elect to divorce on a no-fault basis. In
India, resorting to Khula and Mubarat is a typical method of dissolving a marriage.
Divorce by Mubarat is quite similar to Divorce by Mutual Consent under:
 SECTION 24 of the Special Marriage ACT of 1954
 SECTION 13B of the Hindu Marriage ACT of 1955.

Khula's (At the Request of Wife)


Khula is a word that implies to lie down. It also implies that the husband relinquishes his claim
to and power over his wife. The wife suggests to the husband that the marriage be annulled, and
she can do so by showing some care to him. This might be done by giving up her dower or by
doing something else. This is entirely dependent on whether or not the wife supplies the

43 | P a g e
consideration. So, this sort of divorce begins with the wife, and the husband cannot oppose it, but
he may negotiate the compensation.
In the case of Mst. Bilquis Ikram Vs. Najmal Ikram, it was determined that the woman must
first convince the court that the marriage is pushing her into a terrible union before she is entitled
to Khula.
i. Competence of the parties: The parties must be of sound mind and of majority age.
ii. Free consent: The husband should accept the wife's proposition. On the husband's
behalf, there should be free consent. Force, deception, or other forms of coercion
should not be used to get permission.
iii. Formalities: The woman must make an offer to liberate the husband from the
marriage tie, and the husband must accept the offer.
iv. Consideration: In order to obtain her release, the woman must give something to her
husband. This might be anything from property to money.
Even if the money for consideration is made later, the marriage is dissolved as soon as the
proposal is accepted by the husband.
Mubarat (Mutual Release)
In this scenario, both the husband and the wife are willing to end the marriage. As a result, the
offer of separation might come from either the husband or the wife. The following are the
fundamental aspects of Mubarat divorce:
 Both sides desire to be free of the other.
 There is no need for deliberation because both parties are interested.
The consequences are comparable to those of any other type of divorce. The woman must go
through the iddat period, and the husband must support her throughout that time. Marriage is
dissolved at the end of the time.
1872 Indian Christian Marriage ACT
In India, the provisions of the Indian Divorce ACT of 1869 are applied to Christian marriages.
SECTION X of this statute allows for the dissolution of their marriage. Both partners can seek
for divorce by mutual consent under SECTION XA (as revised in 2001).
The Parsi Marriage and Divorce ACT of 1936 was enacted in 1936
Divorce with Mutual Consent is defined under SECTION 32B. It states that whether the
marriage is solemnized before or after the implementation of the Parsi Marriage and Divorce
(Amendment) ACT, 1988, they can seek for divorce via mutual consent. The suit must be
brought by both parties, and the parties must have been living separately for a year and are
unable to remain together and have jointly chosen to separate. The complaint cannot be brought
until one year has passed from the date of marriage. If the court is satisfied with the facts and
circumstances of the case, the court can issue a divorce decree.

44 | P a g e
The 1954 Special Marriage ACT
In the case of a judicial marriage, the divorce by mutual consent is filed under SECTION 28 of
the Special Marriage ACT, 1954. The parties must jointly submit a petition in court declaring
that they are unable to live together and, as a result, are living separately, and they have both
chosen to file for divorce by mutual consent.
Furthermore, both parties are given a six-month period to consider their choice; nevertheless, if
the parties are adamant about their decision, they can file a motion in court after six months but
before the eighteen-month period expires. When the Court is pleased with the facts and
circumstances of the case, and there are no grounds on which the petition might be dismissed, the
Court can award a divorce order by mutual consent in favour of the parties.

23. Difference between void and voidable marriage under


HMA and legitimacy of children born out of void marriages.

If there are any obstacles (obstructions), the parties will be unable to marry. If someone marries
and there are any impediments in the marriage procedure, the marriage is void. Absolute
obstacles and relative impediments are the two sorts of obstructions.
In absolute obstacles, a truth occurs that disqualifies a person from legitimate marriage, and the
marriage is void from the start, i.e., an invalid marriage from the start.
An impediment that bans marriage with a specific person exists in relative obstacles, and the
marriage is voidable, i.e., one party can escape the marriage. These barriers gave rise to the
marital classifications, which are as follows
i. Void Marriages
ii. Voidable Marriages
A marriage is deemed null and void under the Hindu Marriage ACT if it does not meet the
following standards outlined in SECTION 5 of the Hindu Marriage ACT:
i. Bigamy
If any of the parties has a live spouse at the time of the marriage. It shall be deemed null and
invalid.
In this example, there are three parties: 'A,' 'B,' and 'C.' If 'A' has a live spouse, 'B,' but marries
again to 'C,' this is referred to as bigamy, and the marriage is null and invalid.
ii. Prohibited Degree
If the parties are in an illegal relationship, unless customs authorize it.
As an example, consider the following two parties: 'A' and 'B,' where 'A' is the husband and 'B' is
his wife. They both engaged in a relationship that is illegal under the law. This marriage is also
known as a void marriage.

45 | P a g e
iii. Sapindas
A sapinda marriage, or a marriage between two people who are related to one other or from the
same family.
As an example, consider the following two parties: 'A' and 'B,' where 'A' is the husband and 'B' is
the wife, who has a blood related or a close relation to A, also known as Sapinda. As a result, this
procedure will be considered null and invalid
The following are the effects of a void marriage:
i. In a void marriage, the partners do not have the status of husband and wife.
ii. In a void marriage, children are referred to be legitimate (SECTION 16 of Hindu
Marriage ACT, 1955).
iii. In a void marriage, mutual rights and responsibilities do not exist.
A voidable marriage is one that can be dissolved by any partner. Unless and until a petition to
invalidate the marriage is filed, it will be lawful. According to the Hindu Marriage ACT of 1955,
this marriage must be declared null and void by a competent court. The partners to such a
marriage must determine whether to continue with the marriage or declare it null and void.
The following are the reasons for declaring a marriage void:
i. Due to mental incapacity, the party to the marriage is unable to provide consent.
Illustration: There are two sides, 'A' and 'B,' with 'A' representing the husband and 'B'
representing his wife. 'B' consented to the marriage while suffering from a mental illness. After a
few years, 'B' is cured and raised that her consent was unlawful and that this marriage is voidable
since 'B' was in an unsound mind at the time of her assent. As a result, this is a reason for
voidable marriage.
ii. The party is suffering from a mental illness that renders her unsuited for childbearing.
As an example, consider the following two parties: 'A' and 'B,' where 'A' is the husband and 'B' is
his wife. If 'B' suffers from a mental illness that renders her unsuited for childbearing. Then this
might be a reason for a voidable marriage.
If the party has been subjected to recurrent bouts of lunacy. As an example, consider the
following two parties: 'A' and 'B,' where 'A' is the husband and 'B' is his wife. If someone from
'A' or 'B' suffers from frequent bouts of insanity, this can also be a reason for voidable marriage.
iii. Force or deception is used to get either party's agreement to marry.
As an example, consider the following two parties: 'A' and 'B,' where A is the husband and B is
his wife. If either partner consented to the marriage by coercion or deception, the marriage is
invalid.
iv. If any party is under the age of 21, or the bride is under the age of 18, the marriage
will be voidable.

46 | P a g e
As an example, consider the following two parties: 'A' and 'B,' where 'A' is the husband and 'B' is
his wife. If 'B' is under the age of 18 years, the marriage is voidable, and if A is under the age of
21, the marriage is likewise voidable.
v. If the responder is pregnant with the child of someone other than the bridegroom at
the time of the wedding. As an example, consider the following two parties: 'A' and
'B,' where 'A' is the husband and 'B' is his wife. If 'B' becomes pregnant through
another person during the marriage. The marriage would then be null and invalid.
A petition can be brought before the court on the plea of fraud or application of force on
marriage within one year after the discovery of such fraud or use of force.
The accusation upon which the petition is founded was unknown to the petitioner at the time of
the marriage's solemnization. The petition on such an accusation must be filed in court within
one year of becoming aware of the facts. After learning about the purported facts, no sexual
contact is formed.
SECTION 16 of the Hindu Marriage ACT of 1955 specifies the legitimacy of children born
from invalid and voidable marriages. Any children born out of a void marriage are considered
genuine. Any kid born of a marital connection that is later ruled invalid by a court is also
considered legitimate in a voidable marriage. Even if the marriage is deemed null and invalid
under SECTION 11(void marriage) or SECTION 12, the child born from such marriage is
considered genuine.
If the bride was pregnant prior to the marriage and gave birth to the child after the marriage, that
child cannot be treated as legitimate because that child was not born out of the marital
relationship of the current marriage, and thus the child born after the marriage having been
conceived prior to the marriage is to be held illegitimate. Assume there are two parties, 'A' and
'B,' with 'A' being the husband and 'B' being his wife. 'B' is pregnant through another person
throughout the marriage. The kid born after 'A' and 'B"s marriage does not stem from their
marital bond. That child will be considered illegitimate.

24. "SECTION 125 of Criminal Procedure Code gives rights


of claiming maintenance to wives, children and parents
irrespective of the religion which they profess" Comment.

SECTION 125 of the Civil Procedure Code deals with "Order for support of spouses, children,
and parents." According to SECTION 125(1), the following people may file for and receive
maintenance:
o Wife from his spouse,
o illegitimate or legitimate minor child from his father,
o illegitimate or legitimate minor child (physical or mental abnormalities) from his
father, and

47 | P a g e
o father or mother from his son or daughter.
 Wife
o The Supreme Court defined 'wife' in the case of Chanmuniya v Virendra Singh,
and it encompasses circumstances where a husband and wife have been residing with
each other as husband and wife for a relatively long length of time. Under SECTION
125 of the CrPC, strict evidence of marriage should not be required as a condition to
support.
o The Supreme Court ruled in the particular instance of Smt. Yamunabai Anantrao
Adhav v Ranantrao Shivram Adhav that marriage of a woman in conformance with
Hindu rites with a man who has a living spouse is totally null and void in the eyes of
law, and she is not entitled to benefits under SECTION 125 of the CrPC.
o The Supreme Court ruled in the issue of Sirajmohmedkhan Janmohamadkhan v
Hafizunnisa Yasinkhan that support may be granted to a woman while her husband
is impotent. A woman may claim and receive maintenance from her husband if she is
divorced by her spouse or achieved separation from her husband, has not remarried,
and is unable to support herself.
o Although there is a separate ACT (Muslim Women Protection of Rights on Marriage
ACT) for them, Muslim wives can also claim maintenance under CrPC. A woman
cannot claim and receive support from her husband if she is living in adultery, refuses
to remain with her husband for whatever reason, or lives apart by mutual agreement.
 Minor kid, legitimate or illegitimate
o A 'Minor,' according to SECTION 3 of the Indian Majority ACT of 1875, is a person
who has not acquired his majority, i.e., is above the age of 18 years.
o Under SECTION 125 of the CrPC, a minor son (legitimate or illegitimate) is allowed
to help.
 Daughter
o If a Minor Daughter (Legitimate or Illegitimate) is currently single, she is obligated to
maintenance from her father;
o if she is married, she is also obligated to receive maintenance from her husband's
father; however, the magistrate must be satisfied that her husband lacks essential and
sufficient means to support his minor wife.
o In the instance of Shahbuddin v State of UP, a minor daughter who reached the age of
adulthood during the proceeding of the maintenance claim was found to be eligible to
maintenance up to the date of maturity.
 A legitimate or illegitimate atypical kid who has reached the age of majority
o If a significant kid (legitimate or illegitimate) is abnormal (mentally or physically
unfit), the father of such child is required to support him and may seek maintenance
on this basis.
 Mother or father
o Both the natural father and mother are entitled to support. Adopted mothers are
entitled to maintenance from their adoptive sons. Maintenance is a legal requirement

48 | P a g e
that cannot be evaded by claiming that the father failed to fulfil his parenting
commitment.
 A childless stepmother may be entitled to support.
o The Bombay High Court ruled in Pandurang Bhaurao Dabhade v Baburao Bhaurao
Dabhade that the father or mother might seek maintenance under SECTION 125(1)
(d) if he or she has been unable to support himself or herself.
o However, if parents seek maintenance for their children, the children must have
adequate resources to support their parents while neglecting or refusing to support the
father or mother.

25. OPTIONAL-Extra
26. Registration of marriage, Seema v ashwini kumar,

Facts:
1. The Case arises out of a Petition filed in Haryana District Court regarding the issue of the
registration of marriage which was a matter of the States.
2. Numerous harassment in matrimonial & maintenance cases due to consequences of non
registration of marriages in some states took place.
3. The decision was taken by the court and directions sent to the state to compulsorily register
marriages and report back with the procedure for registration in 3 months.
Issue involved:
 Whether the Marriage registration should be made compulsory in India?
Rationale:
1. The Hindu Law empowers the State Government to draft rules for the registration of
marriages. Under Section 8 (2) of the Hindu Marriage Act, 1955, if the State Government in its
opinion that such registration should be compulsory.
2. Anyone violating rules made in this regard shall be punishable with fine. In spite of the fact
that most of the States have framed rules regarding registration of marriages but still in several
States registration of marriage were not compulsory.
3. The National Commission for Women has specified that non registration of marriages
affects the women. If the marriage is registered it will provide evidence of the marriage having
taken place and would also provide a presumption of the marriage having taken place.
4. The compulsory Registration of Marriage can help in reducing the issue of Child Marriage
which was prevalent at that time. It also provides evidence in the matters of custody of children,
right of children born.

49 | P a g e
Obiter Dicta:
The Supreme Court directs several states to make rules regarding the registration of marriage as
per the following guidelines:-
“Procedure for registration should be reported by respective states within 3 months from the date
of judgement. 
The officer appointed under the said Rules of the States shall be duly authorized to register the
marriages. The age, marital status shall be clearly stated and consequences of non-registration of
marriages shall also be provided in the said Rules to fulfil the purpose of this court. 
When the Central Government enacts a comprehensive statute, the same shall be placed before
this Court for inspection.
Authorised persons for various States and Union Territories shall ensure that the directions given
herein are carried out immediately.”
Judgement:
The Supreme Court upheld the mandatory registration of marriages of all the religion in their
respective States.
Conclusion:
The above stated case deals with the registration of marriages in India. Several states have no
legal rules regarding the registration of marriages.
Non registration of marriages causes several consequences in the form of harassment to women
and child marriage cases.
Referring these issues, the Apex court upheld the mandatory registration of marriages of all the
religion in their respective States and issued certain guidelines for making rules of such
Registration.

27. Conditions and Procedure of registration of marriage


under Special marriage act, 1954.

Irrespective of the rituals followed during the marriage, one can register their marriage under the
Special Marriages Act, by following the procedure below.
For many purposes, (legal formalities, in particular) you might need to do marriage registration
under the Special Marriage Act. The conditions for marriage solemnization under this act differs
in certain aspects when compared to personal laws of religion.
Conditions for Marriage Registration Under the Special Marriage Act
(a) a ceremony of marriage has been performed between the parties and they have been living
together as husband and wife ever since;

50 | P a g e
(b) neither party has more than one spouse living at the time of registration;
(c) neither party is an idiot or a lunatic at the time of registration;
(d) the parties have completed the age of twenty-one years at the time of registration;
(e) the parties are not within the degrees of prohibited relationship;
(f) the parties have been residing within the district of the Marriage Officer, to whom the
application is made for registration of the marriage, for a minimum period of thirty days
immediately prior to the date on which the application is made.
Procedure
The parties to the marriage are required to submit an application
https://services.india.gov.in/service signed by both of them to the Marriage Officer concerned,
along with a marriage affidavit. The Officer will then proceed to give public notice of the
application in the prescribed manner. After allowing a period of thirty days for objection, the
marriage Officer, if satisfied, will enter a certificate of the marriage in the Marriage Certificate
Book, which shall be signed by the parties to the marriage and three witnesses.
From the date of such entry of the marriage certificate, the Marriage will be deemed to be
solemnized under the Special Marriage Act. If the Marriage Officer refuses registration, either
party may appeal to the District Court of the district in which the office of the Marriage Officer
is situated within 30 days of refusal by the Marriage Officer.
Consequences of Marriage Solemnized Under Special Marriage Act
Any member of an undivided family who professes the Hindu, Buddhist, Sikh, or Jain religion
marrying under the Special Marriage Act shall be deemed to affect their severance from the
family. Succession to the property of any person marrying under this Act as well as the
descendants of such person shall be governed by the provisions of the Indian Succession Act,
1925.
However, these rules will not be applicable in case of marriages between a person who professes
the Hindu, Buddhist, Sikh, or Jain religion with a person who professes the Hindu, Buddhist,
Sikh, or Jain religion.

28. Solemnization of marriage under Special marriage act,


1954.

Solemnization of Marriage

After clearing objections, the marriage may be solemnized at the expiry of 30 days, if any field.
The notice is valid for 3 months. Before the marriage is solemnized, the parties and three
witnesses should sign declarations in the prescribed form in the presence of the marriage officer.

51 | P a g e
In whatever form the parties may choose to adopt, marriage can be solemnized. The marriages
can be solemnized either within a reasonable distance from the office of the marriage officer or
at such other place as the parties may wish.

Procedure for solemnisation of marriage

The Special Marriage Act, 1954 of 1954 mandates various preliminaries and civil requirements
before marriage may be solemnised. Both intending parties to the marriage must send a written
notification to the Marriage Registrar of the district in which at least one of the parties to the
marriage has lived for a minimum of 30 days. When the Marriage Registrar obtains the notice of
marriage, he must publish it by affixing a copy to a prominent location in his office. The
Marriage Registrar is required to maintain all notices with records in his office and to register a
genuine copy of each such notice in the ‘Marriage Notice Book,’ which is available to everybody
for examination without charge. Any individual may object to the marriage before the expiration
of thirty days from the date of publication on the grounds that it violates the requirements
established in Section 4 of the Act. 

Following the completion of the thirty-day objection period, the marriage will be solemnised,
unless it has already been opposed to by any individual. In any case, if an objection is raised
against an intended marriage, the Marriage Registrar cannot solemnise the marriage until he has
investigated the matter of objection and reached a decision that the earlier raised objection will
not prevent the marriage from being solemnised, or the prior objection is withdrawn by the
individual raising it. However, if the marriage officer validates the objection and refuses to
solemnise the marriage, any of the intended parties may file an appeal with the district court
within the local limits of the marriage officer’s office within thirty days, and the decision of the
district court on such appeal will be binding, and the marriage officer must act in accordance
with the court’s decision.

Before the marriage may be solemnised, the intended parties and three witnesses must sign a
statement in the prescribed form in the presence of a Marriage Officer, and the declaration must
also be notarized by the Marriage Registrar himself. Following this, the marriage can be
solemnised in the Marriage Registrar’s office or wherever else the parties prefer. In front of the
Marriage Officer and three witnesses, each party must state to the other partner in any language
known by the parties, “I, (X), accept the (Y), to be my lawful wife (or husband).” After the
marriage is solemnised, the Marriage Officer writes the facts on a certificate which he keeps in a
‘Marriage Certificate Book’ also called the Marriage Registration Record which has to be duly
signed by the newlyweds along with the three witnesses and serves as definitive proof of the
marriage. 

Before the expiration of 30 days from the date on which any such notification has been published
under sub-section (2) of Section 6, any person may object to the marriage on the grounds that it
would violate one or more of the requirements mentioned in Section 4.

52 | P a g e
29. Remedies available to a woman when dower is not given.
The right to dower is an inherent right of every Muslim wife. Under Muslim law, following
means of enforcement of the right to dower are available to a wife (or widow):

 Refusal of Conjugal Rights

Before consummation of the marriage, the wife is entitled to deny cohabitation to the husband till
he gives her Prompt Dower on demand. A Muslim-wife can refuse to live with her husband and
refuse to him the sexual intercourse so long as the Prompt Dower is not paid to her.

 Enforcement of Dower as Debt

Where the marriage has been consummated, the wife cannot enforce her claim by refusing
conjugal rights to the husband. In such a situation the wife can recover her unpaid dower by
maintaining an action in a court of law. She may realise it from husband in the same manner as a
creditor recovers his loan.

If the husband dies, the widow is entitled to recover the amount by filing a suit against the legal
heirs of the deceased husband. But the legal heirs of the husband are not personally liable to pay
the dower. The dower is a debt against the estate of the deceased husband which is inherited by
heirs.

 Widow’s Right to Retention

A widow, whose dower remains unpaid, has a right to retain the properties of the husband till her
dower debt is satisfied, whether there is any agreement between the parties for this right or not.

Under this right if a wife has taken possession of her husband’s properties lawfully (with free
consent of the husband) in lieu of unpaid dower, then she is entitled to retain that possession after
the death of her husband, until her dower is paid out of the properties retained by her.

30. Restitution of conjugal rights under muslim, Christian


law and the special marriage act.

Restitution of Conjugal Rights under Muslim Law

The remedy of Restitution of Conjugal Rights is available to Muslims under personal law. Under
Muslim law, the conceptualization of the provision for restitution of conjugal rights can be put
together as follows

53 | P a g e
“Where either the husband or wife has, without lawful ground withdrawn from the society of the
other, or neglected to perform the obligations imposed by law or by the contract of marriage, the
court may decree restitution of conjugal rights, may put either party on terms securing to the
other the enjoyment of his or her rights”

Thus, this concept has been equated with securing the enjoyment of the legal rights of the other
spouse. Previously, this was attached with the specific performance of a contract as well.Unlike
other laws, a suit in a civil court has to be filed and not a petition as per Muslim Law.

When the marriage is valid, only then is a petition for restitution of conjugal rights is
maintainable. Relief of restitution of conjugal rights is an equitable relief and is discretionary.

When without any lawful cause, it is refused by the wife to live with her husband, he can sue for
the restitution of conjugal rights, and similarly, the wife has the right to demand for the
fulfillment of marital duties by the husband. But this is not an absolute right as the Muslim
husband, being the dominant one in matrimonial matters, and as the Quran enjoins the husband
to retain his wife with kindness or to dismiss her with kindness, the Court inclines in support of
the wife and necessitates strict proof of all of the allegations that are necessary for matrimonial
relief.

When can Restitution of Conjugal Rights be refused under Muslim Law?

Defenses for a petition for restitution of conjugal rights could be the grounds of void and
irregular marriages, and other provisions under the Dissolution of Muslim Marriage Act, 1939 .
The suit for restitution of conjugal rights fails when the marriage has been avoided in the
exercise of the option of puberty, under Section 2 (vii) of the Dissolution of Muslim Marriage
Act, 1939.

Controlled polygamy is allowed under Muslim Law, so the comfort consortium to the husband
cannot be refused by the Muslim wife because of the second wife being taken by the husband.
Though, in specific situations, the second marriage of the husband may comprise cruelty to the
first wife, which then justifies her refusal to live with him.

Along with all other instances of cruelty, legal cruelty, as well as physical cruelty, are also
included in the definition of ‘cruelty’ under Section 2 (viii) of the Dissolution of Muslim
Marriage Act, 1939. If any of those instances of cruelty as laid down in this particular section are
proved against the husband, then the relief of restitution of conjugal rights to the husband can be
denied by the Court.

In the case of Itwari v Asghari, where a Muslim husband filed a restitution petition against his
first wife, the Allahabad High Court on 29 August 1959 held that it cannot force the wife to live
with the husband and can refuse the relief if the court feels that it would not be reasonable and
just to do or that passing the decree would be inequitable. Some High Courts have denied the
relief of restitution of conjugal rights on the ground of considering the above as cruelty by the
husband towards the wife.

54 | P a g e
Restitution of Conjugal rights under Christian Law

Under Section 32 and 33 of the Indian Divorce Act, 1869, the remedy of restitution of conjugal
rights is available to Christians.

A petition for restitution of conjugal rights by the husband/wife can be filed before the District
Court or the High Court under Section 32 of the Indian Divorce Act, 1869, when either the
husband or the wife, without reasonable excuse, withdraws himself/herself from the society of
the other. Accordingly, the Court may decree restitution of conjugal rights on being satisfied of
the truth of the statements made in such petition, and there being no legal ground by which the
application should not be granted.

Nothing which would not be a ground for a decree of nullity of marriage or a suit for judicial
separation can be pleaded as a defence against a petition for restitution of conjugal rights under
Section 33 of the Indian Divorce Act, 1869.

Restitution of Conjugal Rights under Special Marriage Act, 1954

The Special Marriage Act is the Act that applies to inter-faith marriages or Court marriages.
Section 22 of the Special Marriage Act 1954 states that when either the husband or the wife has
without reasonable excuse withdrawn from the society of the other, the aggrieved party may
apply by petition to the district court for Restitution of Conjugal Rights.

If the Court is satisfied of the truth of the statements made in the Petition and is assured that
there is no legal ground why the such application should be rejected, it will grant a decree for
Restitution of Conjugal Rights.

31. Definition of aggrieved person and domestic relationship


under law
Section 2 in The Protection of Women from Domestic Violence Act, 2005
2. Definitions.—In this Act, unless the context otherwise requires,—
(a) “aggrieved person” means any woman who is, or has been, in a domestic relationship with
the respondent and who alleges to have been subjected to any act of domestic violence by the
respondent;
(f) “domestic relationship” means a relationship between two persons who live or have, at any
point of time, lived together in a shared household, when they are related by consanguinity,
marriage, or through a relationship in the nature of marriage, adoption or are family members
living together as a joint family;

55 | P a g e
32. Latifi v. Union of India

In Danial Latifi vs. Union of India case, the Court declared that the husband’s liability does not
end with the expiration of Iddat, but that in cases of vagrancy and destitution of the wife, the
husband must maintain her and make reasonable and fair provisions for her even beyond the
customary period.

BENCH: Justice G.B. Pattanaik, Justice S. Rajendra Babu, Justice D.P. Mohapatra, Justice
Doraiswamy Raju, Justice Shivraj V. Patil.

FACTS OF DANIAL LATIFI CASE:

 Shah Bano, a 62-year-old woman from Madhya Pradesh, who was divorced by her
husband in 1978, filed a case for maintenance under Section 125 of the Code of Criminal
Procedure (CrPC).
 The Supreme Court ruled in her favor and upheld the right to alimony for Muslim
women. When the Parliament passed the Muslim Women (Protection of Rights on
Divorce) Act, 1986, which effectively invalidated the decision in the Shah Bano case, she
was denied maintenance.
 Section 3(1) of the Act stated that divorced Muslim women are entitled to reasonable and
fair maintenance during the “Iddat” period, denying divorced wives from their former
husbands subsequent and further maintenance.
 Danial Latifi, the counsel of Shah Bano, viewed the Act passed to be in derogation of
certain provisions of the Constitution because a wife who depended on her husband
before marriage has the right to life even after marriage.
 As a result, this Act infringes on Article 21 of the Constitution. The Act violated Articles
14 and 15 as well as denied divorced Muslim women the same maintenance benefits as
other divorced women under Section 125 of the CrPC.

 Therefore, Daniel Latifi filed a Writ Petition in the Supreme Court challenging the
constitutional validity of the Muslim Women (Protection of Rights on Divorce) Act,
1986.

LEGAL ISSUES:

 Whether Section 3(1) of the Muslim Women (Protection of Rights on Divorce) Act,
1986 inconsistent with Articles 14, 15, and 21 of the Constitution of India?
 Whether the Muslim Women (Protection of Rights on Divorce) Act, 1986
constitutionally sound?

CONTENTIONS:

56 | P a g e
 The petitioner claimed that Section 125 of the CrPC was framed to address a situation in
which a divorced wife was likely to benefit from the divorce, considering Article 21 of
the Indian Constitution, which deals with protecting life and personal liberty.
 If the remedies under section 125 are applied in the case of divorced Muslim women, it
will violate Articles 14, 15, and 21.
 It would be against the secular character of the Constitution if section 125 of the CrPC
were not applied in relation to deprived Muslim women.
 The challenge raised in this petition, according to the Solicitor General, was outside
personal law.

RATIO DECIDENDI:

Justice S. Rajendra Babu: While upholding the constitutional validity of the Muslim Women
(Protection of Rights on Divorce) Act, 1986, the bench observed that under Sec 3(1)(a), a
Muslim husband is responsible for paying maintenance, which may be extended beyond the
Iddat period, as well as making reasonable and fair provisions for his divorced wife’s future.

If a divorced Muslim woman has not remarried and is unable to support herself after the Iddat
period, she can file a claim under Section 4 of the act, which states that she should be supported
by relatives in proportion to the property she leaves to her relatives after her death.

If her relatives cannot support her, the Magistrate may order the State Wakf Board to pay her
support under the Act. The provisions of the Act do not violate Articles 14, 15, and 21 of the
Indian Constitution.

DECISION IN DANIAL LATIFI CASE:

The Constitution Bench of the Supreme Court upheld the view that the wording of Section 3 (1),
“reasonable and fair provision and maintenance to be made and paid to her within the iddat
period by her former husband” was interpreted to mean that the husband must pay maintenance
to the wife before the iddat period expires and that if he cannot do so, the wife may recover it as
provided in Section 3(3) of the Act.

However, nowhere does it state that reasonable and fair provision and maintenance is limited to
the iddat period and not beyond it.

The Court ruled that a husband’s obligation to support and maintain his wife extends for the rest
of the divorced wife’s life unless she marries again.

CONCLUSION:

 The Honorable Supreme Court rendered a balanced verdict in this case by interpreting the
Muslim Women (Protection of Rights on Divorce) Act, 1986 in a manner that saved it
from being struck down as unconstitutional and without compromising personal law or
individual rights.

57 | P a g e
 The Court declared that the husband’s liability does not end with the expiration of Iddat,
but that in cases of vagrancy and destitution of the wife, the husband must maintain her
and make reasonable and fair provisions for her even beyond the customary period.
 Not only did the Court appear progressive in determining women’s rights, but it also
avoided interfering with the Muslim community’s personal laws.
 While many disagree with the verdict given in this case, as far a maintenance dispute is
concerned involving Muslim personal law, the rule in Danial Latifi v. Union of India case
is followed.

33. What are the prohibitions on muslim marriages.


Under the Mohammedan law, Marriage is considered purely as a civil contract and even its
essentials are similar to that of a civil contract, this was observed in the landmark case of Abdul
Kadir v. Salima And Anr.[1] Under this law, the status of the marriage could be divided into
three types-

1. Sahih (Valid)
2. Batil (Void)
3. Fasid (Irregular/ Invalid)

A valid marriage is legitimate while a void marriage has no value in the eyes of law, hence it is
to be assumed two people are not married if the status is void. An irregular marriage is one
which is not valid and has some temporary legal issues, once that obstruction is removed that
irregular marriage becomes valid.

There are generally two types of prohibitions which render a Mohammedan marriage invalid or
void-

1. Absolute Prohibition
2. Relative Prohibition

These prohibitions are not the same and some factors of them can vary for the two major
denominations of Islam, namely Shia and Sunni[2].

Under Shia law, there is no concept of irregular marriage, the Shia law only has either valid or
void marriages; there is no difference in irregular and void marriages. Hence, under Shia law, all
irregular marriages are void. According to Sunni law; there are there statuses of marriage: valid,
invalid and void.

Absolute Prohibition

It is generally considered that a marriage is void if the prohibition regarding it is absolute. There
is no possible method to change the status of marriage from void to valid if the prohibition is
absolute and if someone marries against such prohibitions then that marriage is to be considered
unlawful. Thus marriages prohibited on the ground of consanguinity, affinity, or fosterage are
under absolute prohibition and are void.

58 | P a g e
1. Consanguinity- It means a person is barred to marry with his close blood relatives like
his niece, sister, aunt, etc.
2. Affinity- It means a person is prohibited to marry someone in close relations, for
example, his wife’s daughter, son’s wife, etc.
3. Fosterage- When a lady other than the mother, breastfed a child under the age of two
years, the lady turns to be the foster-mother of the child. A man is restricted from
marrying the persons who come under the foster relationship.

Sunni law has some exceptions regarding the absolute prohibition of fosterage, but Shia law and
jurists deny such exceptions and consider the fosterage the same as consanguinity[3] and
consider the prohibitions to be absolute, all such marriages are rendered void.

Relative Prohibition

These prohibitions are those which render a marriage irregular or invalid. These are considered
to be temporary bars, once these bars are lifted the marriage becomes valid.

All relative prohibitions make a marriage void under Shia law while they render them invalid in
case of Sunni law. Below are some relative prohibitions which can render marriages invalid or
void.

1. Unlawful Conjunction- This bars a man from marrying two different women at the same time
if they are closely related to each other through consanguinity, affinity, or fosterage so that they
could not have lawfully intermarried with each other if they had been of different sexes.

A Muslim man cannot marry a woman who is related to his wife by consanguinity, affinity, or
fosterage, before the death of his wife or divorce.

However, a Shia male can marry his wife’s aunt but to marry her niece, there has to be the
consent of his wife.

2. Marriage with a fifth wife- A Muslim man can marry up to four wives, if he wants to marry
another woman he has to take divorce from one of her four wives. Hence, marriage with five or
more women is unlawful under Islamic laws. Such marriages are invalid or irregular. Under
both, Shia and Sunni laws, polygamy is valid up to four wives. Though it is not encouraged.

However, a Muslim woman cannot marry more than one man. As long as his first husband is
alive or there has been no divorce, she cannot marry a second man. If she marries a second man,
she is guilty of bigamy under section 494 of Indian Penal Code.

3. Absence of witness- Under Sunni law, the presence of a witness is very necessary for the
validation of a marriage, just the presence of guardians is not enough and if there is no proper
witness present, then the marriage itself becomes invalid[4].

59 | P a g e
However, under Shia law, the presence of a witness is not important. A marriage can be valid
even if there is no witness present and marriages can be done in secret as well. But at the time of
the dissolution of a marriage, the presence of a proper witness is required.

4. Differences of religion- Males and females of Shia and Sunni can marry among each other,
however when it comes to religion, a Muslim woman marrying a non-Muslim man is generally
considered to be void, but a Muslim man marrying a non-Muslim woman is considered to be
invalid or irregular[5].

Under Sunni law, males can validly marry non-Muslims of Christianity and Judaism (not
Hinduism) but cannot marry a fire worshipper. However, marriage with fire-worshippers will be
considered irregular and not void. A Shia male and female cannot marry any person of other
religions and all such marriages are considered to be void.

5. Marriage during Iddat- Iddat refers to a period of time which a woman must follow after
getting out of her previous marriage and before remarrying. The time period could vary from one
situation to another. The different time period is defined for the different situation of separations
like divorce, death of husband, etc. Under Sunni law the marriage of a Muslim man with a
woman, who is following the time of iddat is irregular but under Shia law, it is void. The child of
such a marriage will be legitimate if it was born after 6 months, and the mother will be entitled to
dower.

Apart from these, there are various other miscellaneous prohibitions like marriage with a
pregnant woman is void, marriage during a pilgrimage is void, etc. Different jurists have
different opinions on various prohibitions and validation of marriages under the two major
denominations of Islam.

34. Doctrine of factum valet

THE DOCTRINE OF FACTUM VALET:


The maxim “Quod fieri non debris factum valet or the Doctrine of Factum Valet”, is a Latin
maxim, which means ‘what ought not to be done is valid, when done” . After adoption, the child
becomes the aurasa (naturally born son of the adoptive parents) and also gets disqualified to
perform ‘Sradha Karina, Pinda Karma, or Kanyadan' of his natural parents.
In case, there is no other person competent to perform the ceremonies, shastras permit the
adopted son to perform the Sradha Karma , Pinda Karma and Kanyadan. This is known as
‘Factum Valet’. This doctrine was recognised both by the Mitakshara and Dayabhaga
Schools.When there is a rule, which governs an act (not to be done) and the act is done, it cannot
be rendered invalid. This doctrine does not have universal application since every prohibited act,
when done cannot be ratified. Under the Hindu Law, this doctrine gets recognition mainly in the
Law of Marriage and Law of Adoption. Non-compliance of certain. Things relate to a moral
precept, then the violation does not vitiate the proceedings.

60 | P a g e
Eg.:— Marriage overlooking age limit; marriage of a minor without the consent of the guardian.
But where a violation relates to an imperative provision of law' viz, incapacity, non-compliance
of essential ceremonies (Homa, Saptapadi) renders the marriage void and the doctrine does not
apply in such cases. There may be other ceremonies, which are not essential. If they are omitted,
such omission is cured (saved) by this doctrine. The doctrine applies where there is no fraud or
force
ANTE-ADOPTION AGREEMENT
Ante-Adoption Agreement means “An agreement before the Adoption. It is entered into between
the natural parent (person giving I he child in adoption) and the adoptive parent (person taking
the child in adoption) before the adoption in regard to the rights of the adoptive child in the
property of the adoptive parent. It protects the interests of the child (to be given in adoption) in
regard to the property of the adoptive parent. Section 13 of the Hindu Adoptions and
Maintenance Act, 1956 makes provision for the Anti-Adoption Agreement.
Before adoption, the adoptive father, being the sole surviving coparcener, can alienate the
property by will or by a transfer inter vivos (i.e. transfer between the two living persons). If so,
the adoptee (adopted child, who becomes a coparcener after the adoption) cannot question the
alienation by the adoptive lather. If the property alienated is the self-acquired property of the
adoptive father, the adopted child cannot question the alienation even after the adoption.

The difficulty as to the right of alienation of the adoptive father arises, if the property is
ancestral and the alienation is made after the adoption (i.e. the boy acquires the status of
coparcener on adoption. ' the child is a female, she also becomes coparcener in Andhra
Pradesh in pursuance of the A.P. Amendment Act, 1986). To obviate such situation, the
adoptive parent to protect his interests, may resort to enter into an agreement with the
natural parent before taking the child in adoption. Such agreement is called the “ante-
adoption agreement” (designed to protect the interest of the adoptive father).

CASE COMMENT

Problem-1; ‘A’, a Hindu male died in 1953, leaving behind his widow, ‘W’ without
children. ‘W’ inherited the properties of her deceased husband. In 1954, ‘W’ made a gift of
some land from the husband’s property to her grand-niece *N\ which was challenged by
her husband’s collateral, ‘C’. ‘C* sued for possession of the land. The trial court decided in
favour of ‘C \ ‘N’ preferred an appeal. While the appeal was pending, ‘W ’ adopted a son
‘S ’ in 1959 and died in the same year - Decide.
Ans. The instant problem relates to “the Doctrine of Relation Back” from the ‘effects of
adoption’ and attracts Sec. 12(c) of the Hindu Adoptions and Maintenance Act, 1956.The
relevant leading case of the problem is Sawanram vs. Kalawati 1967 SC 1761.The doctrine

61 | P a g e
of Relation Back gives retrospective effect to the adoption. (Write now in brief about the
doctrine of relation back).
According to Sec. 12(c) of the Hindu Adoptions and Maintenance Act, 1956, the adopted child
shall not divert any person of any estate, which rested in him or her before the adoption. Write
now Sawanram’s case in detail as explained under the doctrine of relation back. In view of the
above, the adoption by ‘W’ in 1959, shall have the retrospective effect from the date of the death
of ‘A’. Further, ‘S’ (adopted son) cannot divert any estate, which had vested in ‘W’ prior to his
adoption. Therefore, the gift of the land by ‘W ’ to ‘N’ is valid.
Problem-2: Sujatha, an unmarried woman adopts a daughter, Sudha. Later, Sujatha
married Krishna and has a son by him. Krishna dies leaving the property. Who succeeds
his property? Why?
Clues: Under the Hindu Adoptions and Maintenance Act, 1956, a female child also can be
adopted and an unmarried woman can adopt. Hence, the adoption of Sudha by Sujatha is valid.
(Write (i) the requisites of valid adoption under Sec.6;(ii) the capacity of a female to take in
adoption under Sec.8).
Sudha being the step daughter of the deceased (Krishna) cannot inherit The son, being a Class-I
heir succeeds to the property. (Also write the relevant provisions under the Hindu Succession
Act, 1956 (Ss. 8 to 13).

62 | P a g e

You might also like